Sie sind auf Seite 1von 78

COMMUNITY HEALTH NURSING

1. Which is the primary goal of community health nursing?


a) the promotion of health and prevention of diseases
b) To enhance the capacity of individuals, families and communities to cope with their health
needs
c) Providing services to the people that will increase their level of health
d) To contribute to national development
Ans: b
2. Community health nursing is a community-based practice. Which best explains this
statement?
a) The service is provided in the natural environment of people
b) The nurse has to conduct community diagnosis to find nursing needs and problems
C) The service are based on the available resources within the community
d) Priority setting is based on the magnitude of the health problems identified
Ans:b
3. Population- focused nursing practice requires which of the following processes?
a) Community organizing.
b) Nursing, process
b) Community diagnosis
d) Epidemiologic process
Ans:c
4. What factor must be considered in determining the occupational health privileges to
which the workers will be entitled?
a) Type of occupation: agriculture, commercial, industrial
b) Location of the workplace in relation to health facilities
c) Classification of the business enterprise based on net profit
d) Sex and age composition of employees
Ans:b
5. When the occupational health nurse employs ergonomic principles, she is performing
which roles?
a) Health care provider
b) Health educator
c) Health care coordinator
d) Environment manager
Ans:d
6. Who should provide the occupational health needs of the garment factory does not
have an occupational nurse?
a) Occupational health nurse at the Provincial Health Office
b) Physician employed by the factory
c) Public Health nurse of the RHU of their municipality
d) Rural Sanitary inspector of the RHU in their municipality
Ans:c
7. “Public health services are given free of charge”. Is this statement true or false?
a) The statement is true; it is the responsibility of government to provide haste services
b) The statement is false; people pay indirectly for public health services

Ans:b
8. The most prominent feature of public health nursing is:
a) It involves providing home care to sick people who are not confined in the hospital
b) Services are provided free of charge to people within the catchment area
c) The public health nurse functions as part of a team providing a public health nursing
service
d) Public health nursing focuses on preventive, not curative services
Ans:d
9. According to Margaret Shetland, the philosophy of public health nursing is based on
which of the following?
a) Health and longevity as birthrights
b) The mandate of the state to protect the birthrights of its citizens
c) Public health nursing as a specialized field of nursing
d) The worth and dignity of man
Ans:d
10. Which of the following is the mission of the Department of Health?
a) Health for all
b) Ensure the accessibility and quality of health
c) Improve the general health status of the population
d) Health in the hands of the people by the year 2020
Ans:b
11. Regional Hospital is classified as what level of facility?
a) Primary
b) Secondary
c) Intermediate
d) Tertiary
Ans:d
12. What is true of primary facilities?
a) They are usually government-run
b) Their services are provided on an out-patient basis
c) They are training facilities for health professionals
d) A community hospital is an example of this level of health facilities
Ans:b
13. Which is an example of the school nurse’s health care provider function?
a) Requesting for BCG from the RHU for school entrance immunization
b) conducting random classroom inspection during measles epidemic
b) Taking remedial action on an accident hazard in the school playground
d) Observing places in the school where pupils spend their free times
Ans:b
15. Who is the Chairman of the Municipal Health Board?
a) Mayor
b) Municipal Health Officer
c) Public Health Nurse
d) Any qualified physician
Ans:a
16. Which level of health facility is the usual point of entry of a client into the health
care delivery system?
a) Primary
b) Secondary
c) Intermediate
d) Tertiary
Ans:a
17. Which of the following is a supervisory function of the public health nurse?
a) Referring cases or patients to the midwife
b) providing technical guidance to the midwife
c) Proving nursing care to cases referred by the midwife
d) Formulating and implementing training programs for midwives
Ans:b
18. You are the public health nurse in a municipality with a total population of about
20,000. How many midwife will the RHU need?
a) 1
b) 2
c) 3
d) 4
Ans:d
19. If the RHU needs additional midwife items, you will submit the request for
additional midwife items for approval to the:
a) Rural Health Unit
b) District Health Office
c) Provincial Health Office
d) Municipal Health Board
Ans:d
21. According to Freeman and Heinrich, community health nursing is a developmental
service. Which of the following best illustrates this statement?
a) The community health nurse continuously develops himself personally and professionally
b) Health education and community organizing are necessary in providing community health
services
c) Community health nursing in intended primarily for health promotion and prevention and
treatment of disease.
d) The goal of community health nursing is to provide nursing services to people in their own
place of residence
Ans:b
22. To compare the frequency of the leading causes of mortality in the municipality,
which graph will you prepare?
a) Line
b) Bar
c) Pie
d) Scatter diagram
Ans:b
23. In which step are plans formulated for solving community problems?
a) Mobilization
b) Community organization
c) Follow-up/extension
d) Core group formation
Ans:b
24. The public health nurse takes an active role in community participation. What is the
primary goal of community organizing?
a) To educate the people regarding community health problems
b) To mobilize the people to resolve community health problems
C) To maximize the community’s resources in dealing with health problems
d) None of the above
Ans:d
25. An indicator of success in community organizing is when people are able to:
a) Participate in community activities for the solution of a community problem
b) Implement activities for the solution of the community problem
c) Plan activities for the solution of the community problem
d) Identify the health problem as a common concern
Ans:a
26. Tertiary prevention is needed in which stage of the natural history of disease?
a) Pre-pathogenesis
b) Pathogenesis
c) Predromal
d) Terminal
Ans:d
27. Isolation of a child with measles belongs to what level of prevention?
a) Primary
b) Secondary
c) Intermediate
d) Tertiary
Ans:a
28. Which type of family-nurse contact will provide you with the best opportunity to
observe family dynamics?
a) Clinic consultation
b) Group conferences
c) Home visit
d) written communication
Ans:c
29. Which of the following is an advantage of a home visit?
a) It allows the nurse to provide nursing care to a greater number of people
b) It provides an opportunity to do first hand appraisal of the home situation
c) It allows sharing of experience among people with similar health problems
d) It develops the family’s initiative in providing for health needs of its members
Ans:b
30. Which is contrary to the principles in planning a home visit?
a) A home visit should have a purpose of objective
b) The plan should revolve around the family health .needs
c) A home visit should be conducted in the manner prescribed by RHU
d) Planning of continuing care should involve a responsible-family member
Ans:c
31. The PHN bag is an important tool in providing nursing care during a home visit.
The most important principle in bag technique states that it;
a) Should save time and effort
b) should minimize if not totally prevent the spread of infection
c) should not overshadow concern for the patient and his family
d) May be done in variety of ways depending on the home situation,
Ans:b
32. To maintain the cleanliness of the bag and its contents, which of the following must
the nurse do?
a) Wash his/her hands before and after providing nursing care to the family members
b) In the care of family member’s, as much as possible, use only articles taken from the bag
c) Put on an apron to protect her uniform and fold it with the right side out before putting it
back into the bag.
d) At the end of the visit, fold the lining on which the bag was placed, ensuring that the
contaminated side is on the outside.
Ans:a
33. The public health conducts a study on the factors contributing to the high mortality
rate due to heart diseases in the municipality where she works. Which branch of
epidemiology does the nurse practice in this situation?
a) Descriptive
b) Analytical
c) Therapeutic
d) Evaluation
Ans:b
34. Which of the following is a function of epidemiology?
a) Identifying the disease condition based on manifestations presented by a client
b) Determining factors that contributed to the occurrence of pneumonia in a 3 year old
c) Determining the efficacy of the antibiotic used in the treatment of the 3 year old client with
pneumonia
d) Evaluating the effectiveness of the implementation of the Integrated Management of
Childhood Illness
Ans: d
35. Which of the following is an epidemiologic function of the nurse during an
epidemic?
a) Conducting assessment of suspected cases to detect the communicable diseases
b) Monitoring the condition of the cases affected by the communicable disease
c) Participating in the investigation to determine the source of epidemic
d) Teaching the community on preventive measures against the disease
Ans:c
36. The primary purpose of conducting an epidemiologic investigation is to;
a) Delineate the etiology of the epidemic
b) Encourage cooperation and support of the community
C) Identify groups who are at risk of contracting the disease
d) Identify geographical location of cases of the disease in the community
Ans:a
37. Which is a characteristic of person-to-person propagated epidemic?
a) There are more cases of the disease than expected
b) The disease must necessarily be transmitted through a vector
c) The spread of the disease can be attributed to a common vehicle
d) There is gradual build up of cases before we epidemic becomes easily noticeable
Ans:d
38. In the investigation of an epidemic, you compare the present frequency of the
disease with the usual frequency at this time of the year in this community. This is done
during which stage of the investigation?
a) Establishing the epidemic
b) Testing the hypothesis
c) Formulation of the hypothesis
d) Appraisal of facts
Ans:a
39. The number of cases of Dengue fever usually increases towards the end of the rainy
season. This pattern of occurrence of Dengue fever is best described as;
a) Epidemic occurrence
b) Cyclical variation
c) Sporadic occurrence
d) Secular occurrence
Ans:b
40. Primary health care is a total approach to community development. Which of the
following is an indicator of success in the use of the primary health care approach?
a) Health services are provided free of charge to individuals and families
b) Local officials are empowered as the major decision makers in matters of health
c) Health workers are able to provide care based on identified health needs of the people
d) Health programs are sustained according to the level of development of the community
Ans:d
41. Sputum examination is the major screening tool for pulmonary tuberculosis. Clients
would sometimes get false negative results in this exam. This means that the test is not
perfect in terms of which characteristic of a diagnostic examination?
a) Effectiveness
b) Efficacy
c) Specificity
d) Sensitivity
Ans:d
42. Which of the following demonstrates inter-sectoral linkages?
a) Two-way referral system
b) Team approach
c) Endorsement done by a midwife to another midwife
d) Cooperation between PHN and public school teacher
Ans:d
43. To describe the sex composition of the population, which demographic tool may be
used?
a) Sex ratio
b) Sex proportion
c) Population pyramid
d) Any of these maybe used
Ans:d
44. Which of the following is a natality rate?
a) Crude birth rate
b) Neonatal mortality rate
c) Infant mortality rate
d) General fertility rate
Ans:a
45. You are computing the crude rate of your municipality, with a total population o
about 18,000 for last year. There were 94 deaths. Among those who died, 20 died
because of diseases of the heart and 32 were aged 50 years or older. What is the crude
death rate?
a) 4.1/1000
b) 5.2/1000
c) 6.3/1000
d) 7.3/1000
Ans:b
46. Knowing that malnutrition is a frequent community health problem, you decided to
conduct nutritional assessment. What population is particularly susceptible to protein
energy malnutrition (PEM)?
a) Pregnant women and the elderly
b) Under 5 year old children
c) 1-4 year old children
d) School age children
Ans:c
47. Which statistic can give the most accurate reflection of the health status of a
community?
a) 1-4 year old age-specific mortality rate
b) Infant mortality rate
c) Swaroop’s index
d) Crude death rate
Ans:c
48. In the past year, village A had an average population of 1655. 46 babies were born
in that year, 2 of whom died less than 4 weeks after they were born. They were 4
recorded stillbirths. What is the neonatal mortality rate?
a) 27.8/1000
b) 43.5/1000
c) 86.9/1000
d) 130.4/1000
Ans:b
49. Which statistic best reflects the nutritional status of a population?
a) 1-4 year old age-specific mortality rate
b) Proportionate mortality rate
c) Infant mortality rate
d) Swaroop’s index
Ans:a
50. What numerator is used in computing general fertility rate?
a) Estimated midyear population
b) Number of registered live births
c) Number of pregnancies in the year
d) Number of females of reproductive age
Ans:b
51. You will gather data for nutritional assessment of a purok. You will gather
information only from families with members who belong to the target population for
PEM. What method of data gathering is best for this purpose?
a) Census
b) Survey
c) Record Review
d) Review of civil registry
Ans:b
52. Which criterion in priority setting of health problems is used only in community
health care?
a) Modifiability of the problem
b) Nature of the problem presented
c) Magnitude of the health problem
d) Preventive potential of the health problem
Ans: c
53. Which of the following women should be considered as special targets for family
planning?
a) Those who have two children or more
b) Those with medical conditions such as anemia
c) Those younger than 20 years and older than 35 years
d) Those who just had a delivery within the past 15 months
Ans:d
54. A woman, 6 months pregnant, came to the center for consultation. Which of the
following substances is contraindicated?
a) Tetanus toxoid
b) Retinol 200,000 IU
c) Ferrous sulfate 200mg
d) Potassium iodate 200 mg, capsule
Ans:b
55. During prenatal consultation, a client asked you if she can have her delivery at
home. After history taking and physical examination, you advised her against a home
delivery. Which of the following findings disqualifies her for a home delivery?
a) Her OB score is G5P3
b) She has some palmar pallor
c) Her blood pressure is 130/80
d) Her baby is in cephalic presentation
Ans:a
56. Inadequate intake by the pregnant woman of which vitamin may cause neural tube
defects?
a) Niacin
b) Riboflavin
c) Folic Acid
d) Thiamine
Ans:c
57. You are in a client’s home to attend to a delivery. Which of the following will you do
first?
a) Set up a sterile area
b) Put on a clean gown and apron
c) Cleanse the client’s vulva with soap and water
d) Note the interval, duration and intensity of labor and contractions
Ans:d
58. In preparing a primigravida for breastfeeding, which of the following will you do?
a) Tell her that lactation begins within a day after delivery
b) Teach her nipple stretching exercises if her nipples are everted
c) Instruct her to wash her nipples before and after each breastfeeding
d) Explain to her that putting the baby to breast will lessen blood loss after delivery
Ans:d
59. A primigravida is instructed to offer her breast to the baby for the first time within
30 minutes after delivery. What is the purpose of offering the breast this early?
a) To initiate the occurrence of milk letdown
b) To stimulate milk production by the mammary acini
c) To make sure that the baby is able to get the colustrum
d) To allow the woman to practice breastfeeding in the presence of the health worker
Ans:b
60. In a mother’s class, you discuss proper breastfeeding technique. Which of these is a
sign that the baby has “lactated on” the breast property?
a) The baby takes shallow, rapid sucks
b) The mother does not feel nipple pain
c) The baby’s mouth is only partly open
d) Only the mother’s nipple is inside the baby’s mouth
Ans: b
61. You explain to a breastfeeding mother that breastmilk is sufficient for all of the
baby’s nutrient needs only up to:
a) 3 months
b) 6 months
c) 1 year
d) 2 years
Ans:b
62. What is given to a woman within a month after the delivery of a baby?
a) Malunggay capsule
b) Ferrous sutfate l00mg O.D)
c) Retinol 200.000 IU 1 capsule
d) Potassium Iodate 200 mg, 1 capsule
Ans:c
63. Which biological used in EPI is stored in the freezer?
a) DPT
b) Tetanus toxoid
c) Measles vaccine
d) Hepatitis B vaccine
Ans:c
64. Unused BCG should be discarded how many hours after reconstitution?
a) 2
b) 4
c) 6
d) At the end of the day
Ans:b
65. Which immunization produces a permanent scar?
a) DPT
b) BCG
c) Measles vaccination
d) Hepatitis B vaccination
Ans:b
66. A 4 week old baby was brought to the health centre for his first immunization.
Which can be given to him?
a) DPT1
b) OPV1
c) Infant BCG
d) Hepatitis B Vaccine
Ans:c
67. You will not give DPT 2 if the mother says that the infant had?
a) Seizures a day after DPT1
b) Fever for 3 days after DPT1
c) Abscess formation after DPT1
d) Local tenderness for 3 days after DPT1
Ans:a
68. Polio vaccines stored in the refrigerator at the temperature of
a) -40o C
b) -20oC
c) -30oC
d) -10oC
Ans: b
69. PPD stands for
a) Pulse polio device
b) Protein pure devices
c) Purified protein derivatives’
d) Purified polio derivates
Ans:c
70. “Health for all act” was proposed in
a) 1976
b) 1977
c) 1942
d) 1988
Ans: b
71. Which of the following signs will indicate that a young child is suffering from severe
pneumonia?
a) Dyspnea
b) Wheezing
c) Fast breathing
d) Chest indrawing
Ans:d
72. Using IMCI guidelines, you classify a child as having severe pneumoniA) What is
the best management for the child?
a) Prescribe antibiotic
b) Refer him urgently to the hospital
c) Instruct the mother to increase fluid intake
d) Instruct the mother to continue breastfeeding
Ans:b
73. When small pox eradicated from India
a) 1910
b) 1950
c) 1975
d) 1965
Ans:c
74. CHC covers
a) 200-300 bed
b) 30-50 bed
c) 10-15 bed
d) None of these
Ans: b
75. Old name of under fivce clinic is
a) Child clinic
b) Well baby clinic
c) Health baby clinic
d) Under five clinic
Ans:b
76. The beginning of 1st five year plan is in
a) 1952
b) 1956
c) 1951
d) 1976
Ans: c
77. International womens day is celebrated in
a) 10th apri;l
b) 8th march
c) 2nd august
d) 9th june
Ans:b
78. during the physical examination of a young child, what is the earliest sign of
xerophthalmia that may observe?
a) Keratomalacia
b) Corneal opacity
c) Night blindness
d) Conjunctival xerosis
Ans:d
79. To prevent xerophthalmia, young children are given Retinol capsule every 6 months.
What is the dose given to preschoolers?
a) 10, 000 IU
b) 20, 000 IU
c) 100, 000 IU
d) 200, 000 IU
Ans:d
80. The major sign of iron deficiency anemia is pallor. What part is best examined for
pallor?
a) Palms
b) Nailbeds
c) Around the lips
d) Lower conjunctival sac
Ans:a
81. Food fortification is one of the strategies to prevent micronutrient deficiency
conditions. RA 8976 mandates fortification of certain food items. Which of the following
is among these food items?
a) Sugar
b) Bread
c) Margarine
d) Filled milk
Ans:a
82. What is the best course of action when there is a measles epidemic in a nearby
municipality?
a) Give measles vaccine to babies aged 6 to 3 months
b) Give babies aged 6 to 11 months one dose of 100,000 IU of Retinol
c) Instruct mother to keep their babies at home to prevent disease transmission
d) Instruct mothers to feed their babies adequately to enhance their babies resistance
Ans:a
83. World hepatitis day celebrated in
a) 5th april
b) 16th july
c) 28 july
d) 18 june
Ans: c
84. Management of a child with measles includes the administration of which of the
following?
a) Gentian violet on mouth lesions
b) Antibiotic to prevent pneumonia
c) Tetracycline eye ointment for corneal opacity
d) Retinol capsule regardless of when the last dose was given
Ans:d
85. Bharat sevak samaj formed in
a) 1952
b) 1962
c) 1972
d) 1982
Ans:a
86. The all india blind relief society established in
a) 1946
b) 1948
c) 1950
d) 1920
Ans: a
87. Secondary prevention for malaria includes?
a) Planting of neem or eucalyptus trees
b) Residual spraying of insecticides at night
c) Determining whether a place is endemic or not
d) Growing larva-eating fish in mosquito breeding places
Ans:c
88. Scotch tape swab is done to check for which intestinal parasite?
a) Ascaris
b) Pinworm
c) Hookworm
d) Schistosoma
Ans:b
89. Which of the following signs indicates the need for sputum examination for AFB?
a) Hematemesis
b) Fever for 1 week
c) Cough for 3 weeks
d) Chest pain for 1 week
Ans:c
90. Which clients are considered targets for DOTS category?
a) Sputum negative cavitary cases
b) Clients returning after default
c) Relapses and failures of previous PTB treatment regimens
d) Clients diagnosed for the first time through a positive sputum exam
Ans:d
91. To improve compliance to treatment, what innovation is being implemented in
DOTS?
a) Having the health worker follow up the client at home
b) Having the health worker or a responsible family member monitor drug intake
c) Having the patient come to the health center every month to get his medications
d) Having a target list to check on whether the patient has collected his monthly supply of
drugs
Ans:b
92. Diagnosis of leprosy is highly dependent on recognition of symptoms. Which of the
following is an early sign of leprosy?
a) Macular lesions
b) Inability to close eyelids
c) Thickened painful nerves
d) Sinking of the nose bridge
Ans:c
93. Which of the following clients should be classified as a case of mutibacillary leprosy?
a) 3 skin lesions, negative slit skin smear
b) 3 skin lesions, positive slit skin smear
c) 5 skin lesions, negative slit skin smear
d) 5 skin lesions, positive slit skin smear
Ans:d
94. For prevention of Hepatitis A, you decided to conduct health education activities.
Which of the following is Irrelevant?
a) Use of sterile syringes and needles
b) Safe food preparation and food handling by vendors
c) Proper disposal of human excreta and personal hygiene
d) Immediate reporting of water pipe leaks and illegal water connections
Ans: a
95. Which biological used in EPI should not be stored in the freezer?
a) DPT
b) OPV
c) Measles vaccine
d) MMR
Ans: a
96. In IMCI, severe conditions generally require urgent referral to a hospital. Which of
the following severe conditions Does not always require urgent referral to hospital?
a) Mastoiditis
b) Severe dehydration
c) Severe pneumonia
d) Severe febrile disease.
Ans:b
97. A client was diagnosed as having Dengue Fever. You will say that there is slow
capillary refill when the color of the nailbed that you pressed does not return within
how many seconds?
a) 3
b) 5
c) 8
Dd) 10
Ans:a
98. Bhopal gas tradegy was occurred in
a) 1986
b) 1984
c) 1987
d) 1988
Ans: b
99. The pathognomonic sign of measles is Koplik’s spot. You may see Koplik’s spot by
inspecting the:
a) Nasal Mucosa
b) Buccal mucosa
c) Skin on the abdomen
d) Skin on the antecubital surface
Ans:b
100. Among the following diseases, which is airborne?
a) Viral conjunctivitis
b) Acute poliomyelitis
c) Diphtheria
d) Measles
Ans:d
101. Among children aged 2 months to 3 years, the most prevalent form of meningitis is
caused by which microorganism?
a) Hemophilus Influenzae
b) Morbillivirus
c) Streptococcus Pneumoniae
d) Neisseria meningitides
Ans:a
102. Balwadi nutritional program was launched in
a) 1970
b) 1968
c) 1960
d) 1946
Ans: a
103. Human beings are the major reservoir of malaria, Which of the following strategies
in malaria control is based on this fact?
a) Stream seeding
b) Stream clearing
c) Destruction of breeding places
d) Zooprophylaxis
Ans:d
104. The use of larvivorous fish in malaria control is the basis for which strategy of
malaria control?
a) Stream seeding
b) Stream clearing
c) Destruction of breeding places
d) Zooprophylaxis .
Ans:a
105. Mosquito-borne diseases are prevented mostly with the use of mosquito control
measures. Which of the following is NOT appropriate for malaria control?
a) Use of chemically treated mosquito nets
b) Seeding of breeding places with larva-eating fish
c) Destruction of breeding places of the mosquito vector
d) Use of mosquito-repelling soaps, such as those with basil or citronella
Ans:c
106. The national water supply and sanitation programme was launched in
a) 1954
b) 1911
c) 1986
d) 1981
Ans: a
107. Applied nutrional program was launched in
a) 1923
b) 1963
c) 1943
d) 1953
ans: b
108. Which is the BEST control measure for AIDS?
a) Being faithful to a single sexual partner
b) Using a condom during each sexual contact
c) Avoiding sexual contact with commercial sex workers
d) Making sure that one’s sexual partner does not have signs of AIDS
Ans:a
109. The most frequent causes of death among clients with AIDS are opportunistic
diseases. Which of the following opportunistic infections is characterized by
tonsilllopharyngitis?
a) Respiratory candidiasis
b) Infectious mononucleosis
c) Cytomegalovirus disease
d) Pneumocystis carinii pneumonia
Ans:b
110. Number of district in india
a) 612
b) 614
c) 116
d) 671
Ans:d
111. Number of state and union territory in india
a) 28, 7
b) 30, 5
c) 29, 6
d) 18, 19
Ans: a

112. The commonest constituent naturaly present in water


a) Iodine
b) Chlorine
c) Iron
d) Fluorine
Ans: d
113. A good health triad does not include:
a) Physical Status
b) Social Status
c) Mental Status
d) Economic Status
Ans: d
114. Determinants of Health does not include:
a) Host
b) Agent
c) Risk Factors
d). Environment
Ans: b
115. Epidemiological triad does not include:
a) Host
b) Agent
c) Risk factors
d) Environment
Ans: c
116. According to Gordon’s classification of prevention of disease includes all except
one:
a) Primary prevention
b) Universal prevention
c) Selective prevention
d) Indicated prevention
Ans: a
117. Prevent complications is included in which level of prevention ?
a) Primary
b) Primordial
c) Secondary
d) Tertiary
Ans: c
118. Epidemiology does not include which study?
a) Frequency
b) Distribution
c) Description
d) Determination

Ans: c
119. New cases in population during a fixed period is determined by :
a) Incidences Rates
b) Prevalence Ratio
c) Attack Rate
d) Disability Rates

Ans: a
120. Cross sectional study is included in which type of epidemiology study?
a) Analytical
b) Descriptive
c) Randomised control
d) Ecological study
Ans: a
121. Advantages of cohort study are all except:
a) Incidence cannot be calculated
b) Bias minimised
c) Relative risk can be estimated
d) Dose response ratio can be calculated
Ans: a
122. Study which proceed from effect to cause:
a) Cohort
b) Case Control
c) Ecological
d) Experimental
Ans: b
123. Severity of epidemic does not depend upon:
a) Economic Conditions
b) Host Population
c) Geographic Conditions
d) Social-cultural Behaviour
Ans: b
124. 1st step of investigation of epidemic is:
a) Confirmation of existence
b) Varification of diagnosis
c) Analysis of data
d) Search the cases
Ans: b
125. Which is not the step involved in preparedness of an epidemic :
a) Manpower training
b) Logistics
c) Rehabilitation
d) Diagnosis and Treatment
Ans: b
126. Which is not a type of screening:
a) Mass Screening
b) Multiple Screening
c) Low risk Screening
d) High risk Screening
Ans: c
127. Capability to identify who has the disease is done by measuring :
a) Validity
b) Sensitivity
c) Specificity
d) Repeatability
Ans: b
128. Screening test is not useful when?
a) Incidence of the disease
b) Incidence is low in the community
c) Early detection leads to favorable outcome
d) The disease has a lead time
Ans:B
129. Provision of free medical care to the people at government expense is known as?
a) State medicine
b) Social therapy
c) Social medicine
d) Social insurance programme
Ans:A
130. Nutritional status of children between 0-4 years in a community can be assessed by
all except?
a) Mortality in 0-4 years
b) Birth weight of less than 2.5 gm
c) Maternal Hb<11.5 g/dl/
d) Height and weight of all preschool children
Ans:c
131. Population of 10000, birth rate 36 per 1000, 5 maternal deaths, the MMR is?
a) 14.5
b) 13.8
c) 20
d) 5
Ans:b
132. Which of the following is not in WHO surveillance?
a) Rabies
b) Influenza
c) Malaria
d) Varicella
Ans:d
133. The relationship between incidence and prevalence can be expressed as the?
a) Product of incidence and mean duration of disease
b) Dividends of incidence and mean duration of disease
c) Sum of incidence and mean duration of disease
d) Difference of incidence and mean duration of disease
Ans:a
134. A nurse is planning a community education presentation about testicular cancer.
The target group should be men aged
a) 20 to 34 years
b) 40 to 49 years
c) 50 to 64 years
d) 65 years and older
Ans:a
135. The last case of smallpox was reported in the world in:
a) 1977
b) 1978
c) 1979
d) 1982
Ans:a
136. Smallpox was eradicated because of the following factors except:
a) Potent vaccine
b) Easy to recognize
c) Extra human reservoir control
d) Long incubation period
Ans:c
137. Which of the following is true of chickenpox?
a) Virus is not found in scab =
b) Virus can be grown on the chick embryo
c) Caused by RNA virus
d) Does not cross the placental barrier
Ans:a
138. Infectivity of chickenpox last
a) Till the last scab fall offs
b) 3 days after appearance of rash
c) 6 days after appearance of rash
d) As long as fever last
Ans:c
139. All stages of rash are seen in:
a) Chickenpox
b) Smallpox
c) Measles
d) Typhoid
Ans:a
140. Carriers are associated with transmission of disease in all except:
a) Typhoid
b) Cholera
c) Measles
d) Diphtheria
Ans:c
141. Koplik's spots are seen in:
a) Rubella
b) Rubeola
c) Typhoid
d) Chickenpox
Ans:b
142. All are true about measles except:
a) Incubation period of 10 to 14 days
b) Secondary attack rate of 30%
c) More severe in malnourished
d) Sub clinical infection
Ans:b
143. Sub acute sclerosing panencephalitis (SSPE) can occur in infection with:
a) Measles
b) Rubella
c) Mumps
d) Chickenpox
Ans:a
144. Which of the following diseases gives life-long immunity after an attack?
a) Mumps
b) Typhoid
c) Tetanus
d) Diphtheria
Ans:b
145. Mycobacterium tuberculosis and Vibrio cholerae were identified by:
a) Van Helmholtz (1843)
b) Robert Koch (1843-1910)
c) Louis Pasteur (1822-1895)
d) Van Leeuwenhoek (1683),
Ans:b
146. The bacillus that causes plague is called)
a) Yersinia pestis
b) Fusobacterium clostridiformis
c) S. aureus
d) Perfringens
Ans:a
147. Period of maximum infectivity in mumps is during
a) incubation period
b) onset of swelling
c) after swelling appears
d) prodromal period
Ans:b
148. Shick test is done in:
a) meningitis
b) diphtheria
c) pertussis
d) polimyelitis
Ans:b
149. Recommended site of administration of DPT in infants is
a) gluteal
b) deltoid
c) lateral aspect of thigh
d) forearm
Ans:b
150. . Major epidemics of influenza A occur at intervals of:
a) 1 year
b) 2-3 years
c) 10-15 years
d) 20-25 years
Ans:d
151. The vaccine usually given to pilgrims to the Middle East is:
a) DPT
b) influenza
c) yellow fever
d) meningitis
Ans:c
152. Measles vaccine is given under the National Immunization Programme:
a) 6 weeks
b) 6 months
c) 9 Months
d) 18 months
Ans:c
153. Modifiable risk factors in case of Hypertension include all, EXCEPT:
a) Age
b) Obesity
c) Salt intake
d) Alcohol intake
Ans:a
154. Infections primarily of animals that can be transmitted through milk is all,
EXCEPT:
a) Tuberculosis
b) Brucellosis
c) Q fever
d) Malaria
Ans:d
155. The diseases notifiable to the WHO under the International Regulation Act are all
except:
a) Cholera
b) Plague
c) Yellow Fever
d) Diphtheria
Ans:d
156. Quickening means:
a) Hearing of the fetal heart sound
b) Sudden cessation of Menstruation
c) The movement of the feuts in the uterus felt by the mother
d) Nausa and vomiting in the morning
Ans:d
157. Discolouration of teeth is caused by a kind of antibiotics called)
a) Streptomycin
b) Neomycin
c) Tetracycline
d) Penicillin
Ans:c
158. If a child with diarrhoea registers two signs in the yellow row in the IMCI chart,
we can classify the patient as:
a) Moderate dehydration
b) Some dehydration
c) Severe dehydration
d) No dehydration
Ans: b
159. Indian council for child welfare was established in
a) 1952
b) 1962
c) 1972
d) 1982
Ans: b
160. A child who is 7 weeks has had diarrhoea for 14 days but has no sign of
dehydration is classified as?
a) Persistent diarrhoea
b) Severe dysentery
c) Dysentery
d) Severe Persistent diarrhoea

Ans: d
161. The child with no dehydration needs home treatment. Which of the following is not
included in the rules for home treatment in this case?
a) Forced fluids
b) When to return
c) Give Vitamin A supplement
d) Feeding more

Ans: c
161. Fever as used in IMCI includes:
a) Axillary temperature of 37.5 or higher
b) Rectal temperature of 38 or higher
c) Feeling hot to touch
d) All of the above
e) A and C only

Ans: d
162. An important role of the community health nurse in the prevention and control of
Dengue Hemorrhagic-fever includes:
a) Advising the elimination of vectors by keeping water containers covered
b) Conducting strong health education drives/campaign directed toward proper garbage
disposal
c) Explaining to the individuals, families, groups and community the nature of the
disease and its causation.
d) Practicing residual spraying with insecticides

Ans: c
163. Community health nurses should be alert in observing a Dengue suspect. The
following is NOT an indicator for hospitalization of H-fever suspects?
a) Marked anorexia, abdominal pain and vomiting
b) Increasing hematocrit count
c) Cough of 30 days
d) Persistent headache

Ans: c
164. The community health nurse’s primary concern in the immediate control of
haemorrhage among patients with dengue is:
a) Advising low fiber and non-fat diet
b) Providing warmth through light weight covers
c) Observing closely the patient for vital signs leading to shock
d) Keeping the patient at rest.

Ans: c
165. Which of these signs may NOT be REGARDED as a truly positive signs indicative
of Dengue H-fever?
a) Prolonged Bleeding Time
b) Appearance of at least 20 petechiae within 1 cm square
c) Steadily increasing hematocrit count
d) Fall in the platelet count

Ans: b
166. Which of the following is the most important treatment of patients with Dengue H-
fever?
a) Give aspirin for fever
b) Replacement of body fluids
c) Avoid unnecessary movement
d) Ice cap over abdomen in case of Melina

Ans: b
167. In correcting misconception and myths about certain diseases and their
management, the health worker should first:
a) Identify the myths and misconceptions prevailing in the community
b) Identify the source of these myths and misconceptions
c) Explain how and why these myths came about
d) Select the appropriate IEC strategies to correct them.

Ans: a
168. How many percent of measles are prevented by immunization at 9 months age?
a) 85%
b) 90%
c) 99 %
d) 95 %

Ans: a
169. After TT3 vaccination a mother is said to be protected to tetanus by around?
a) 80 %
b) 85 %
c) 99 %
d) 95 %

Ans: d
170. If ever convulsion occurs after administering DPT, what should nurse best suggest
to the mother?
a) Do not continue DPT vaccination anymore
b) Advise mother to come back aster 1 week
c) Give DT instead of DPT
d) Give pertussis of the DPT and remove DT
Ans: c
171. These vaccines are given 3 doses at one month intervals:
a) DPT, BCG, TT
b) DPT, TT, OPV
c) OPV, Hep. B, DPT
d) Measles, OPV, DPT

Ans: c
172. Which among the following is the primary focus of prevention of cancer?
a) Elimination of conditions causing cancer
b) Diagnosis and treatment
c) Treatment at early stage
d) Early detection

Ans: a
173. In the prevention and control of cancer, which of the following activity is the most
important function of the community health nurse?
a) Conduct community assemblies
b) Referral to cancer specialist those clients with symptoms of cancer
c) Use the nine warning signs of cancer as parameters in our process of detection;
control and treatment modalities.
d) Teach woman about proper/correct nutrition.
Ans: a
174. Who among the following are recipients of the secondary level of care for cancer
cases?
a) Those under early case detection
b) Those under supportive care
c) Those scheduled for surgery
d) Those under going treatment

Ans: a
175. Who among the following are recipients of the tertiary level of care for cancer
cases?
a) Those under early treatment
b) Those under supportive care
c) Those under early detection
d) Those scheduled for surgery

Ans: b
176. In Community Health Nursing, despite the availability and use of many equipment
and devices to facilitate the job of the community health nurse, the nurse should be
prepared to apply is a scientific approach. This approach ensures quality of care even at
the community setting. This nursing parlance is nothing less than the:
a) Nursing diagnosis
b) Nursing protocol
c) Nursing research
d) Nursing process

Ans: d
177. The average amount of breast miulkk secreted by an Indian women is
a) 200-400ml
b) 450-600ml
c) 500-800ml
d) 100-200ml
Ans:b
178. Why are small for gestational age newborns at risk for difficulty maintaining body
temperature?
a) their skin is more susceptible to conduction of cold
b) they are preterm so are born relatively small in size
c) they do not have as many fat stored as other infants
d) they are more active than usual so they throw off comes

Ans: c
179. Oxytocin is administered to Rinna to augment labor. What are the first symptoms
of water intoxication to observe for during this procedure?
a) headache and vomiting
b) a swollen tender tongue
c) a high choking voice
d) abdominal bleeding and pain

Ans:a
180. Which of the following treatment should NOT be considered if the child has severe
dengue hemorrhagic fever?
a) use plan C if there is bleeding from the nose or gums
b) give ORS if there is skin Petechiae, persistent vomiting, and positive tourniquet test
c) give aspirin
d) prevent low blood sugar

Ans: c
181. In assessing the patient’s condition using the Integrated Management of Childhood
Illness approach strategy, the first thing that a nurse should do is to:
a) ask what are the child’s problem
b) check the patient’s level of consciousness
c) check for the four main symptoms
d) check for the general danger signs

Ans: a
182. A child with diarrhea is observed for the following EXCEPT:
a) how long the child has diarrhea
b) skin Petechiae
c) presence of blood in the stool
d) signs of dehydration

Ans: b
183. The child with no dehydration needs home treatment. Which of the following is
NOT included in the care for home management at this case?
a) give drugs every 4 hours
b) continue feeding the child
c) give the child more fluids
d) inform when to return to the health center

Ans: a
184. Ms. Jordine, RN, believes that a patient should be treated as individual. This
ethical principle that the patient referred to:
a) beneficence
b) nonmaleficence
c) respect for person
d) autonomy

Ans: c
185. When patients cannot make decisions for themselves, the nurse advocate relies on
the ethical principle of:
a) justice and beneficence
b) fidelity and nonmaleficence
c) beneficence and nonmaleficence
d) fidelity and justice

Ans: c
186. Institutional hospiutal covers
a) 100-200beds
b) 200-300beds
c) 400-500beds
d) 500 and above
Ans: d
187. What is one way a nurse might bring population assessment into play
a) Make every patient a case for personal care
b) Work as a volunteer in a wildlife organization
c) Apply for a grant to provide health care for a specific population.
d) Focus on a standard formula)
Ans: c
188. The Healthy Cities movement focuses on
a) providing hospitals in urban areas.
b) promoting health and preventive health in urban areas.
c) funding city governments to demolish old housing.
d) moving residents out of areas in the city center.
Ans:b
189. Which of the following methods may help a community health nurse gain entry
into a community?
a) Making clear who is in charge
b) knocking on doors
c) joining certain clubs or societies
d) aligning mainly with leaders of the group
Ans: c
190. Community diagnoses are based on
a) The community assessment process
b) the intuition of the skilled community nurse
c) a single method, without bias
d) court records
Ans: a
191. Primary prevention activities include one of the following.
a) Retraining and education to maximize remaining capacities
b) prompt treatment after early diagnosis
c) treatment to arrest the disease process
d) health education
Ans:d
192. Which of the following is an example of high-level wellness?
a) following an imposed physical training regimen for someone else's benefit
b) managing one's chronic illness while living a happy life, doing the most one can
c) recovering quickly from a broken arm
d) keeping up a level of perfect physical health
Ans:b
193. What factor among the following can influence food choice?
a) Convenience foods
b) lack of exercise
c) distance from grocery store
d) mortality statistics
Ans:c
194. What cognitive factor has been shown to be important to clients' health-promoting
behaviors?
a) Access to care
b) demographics
c) environment
d) control
Ans:a
195. in health education theory, what is a barrier?
a) what prevents a patient from achieving a health-enhancing behaviour
b) a patient's conviction of self-efficacy
c) belief on the nurse's part that a patient should, "Do it right, or not at all." a focus on
d) disease prevention
Ans:a

196. The causative organism for dengue fever is


a) Enteric virus
b) Arbovirus
c) Variola virus
d) Influenmza virus
Ans: b
197. Poliomyelitis is transmitted through
a) Water
b) Air
c) Blood
d) Vector
Ans: a
198. Ideas’ national; health policy was approved in the parliament in
a) 1961
b) 1980
c) 1983
d) 1991
Ans: c
199. A mechanical method of contraception
a) Vasectomy
b) Mala-D
c) Condom
d) MTP
Ans:c
200. Kartar singh committee is otherwise known as
a) Multipurpose workers committee
b) Health survey and development committee
c) Health survey and planning committee
d) Committee on medical education
Ans: a
201. The desired quantity of the free residual chlorine in the drinking water is
a) 1mg/lit
b) 0.5mg/lit
c) 2mg/lit
d) 0.25mg/lit
Ans: b
202. The hormonal contraceptives are
a) Oral
b) injectable
c) Implant
d) all of the above
Ans: d
203. MCH service includes
a) prenatal and post natal care
b) under five care
c) family planning service
d) all of them
Ans: d
204. The recommended maximum noise level for human beings
a) 85 db
b) 50 db
c) 110db
d) 160db
Ans: a
205. The best method of refuse disposal in hospital is
a) Dumping
b) Controlled tipping
c) Incineration
d) Composting
Ans: c
206. MTP act was passed in the year
a) 1983
b) 1977
c) 2000
d) 1972
Ans: d
207. The average amount of blood loss during normal delivery
a) 300ml
b) 100ml
c) 500ml
d) 700ml
Ans: c
208. Which of the following is not a occupational disease
a) Asbestosis
b) Pneumoconiosis
c) Japanese enchapilities
d) Silicosis
Ans: c
209. Causative organism for malaria is
a) Cryptococcus
b) Leishmania
c) Plasmodium
d) Klebsiella
Ans: c
210. World no tobacco day is observed on
a) 5th JUNE
b) 7th april
c) 10th August
d) 31st may
Ans:d
212. World heart day is observed in
a) 21 Seb
b) 28 Sep
c) 10 Oct
d) 14 Aug
Ans: b
213. Permanent male sterilization is
a) Vasectomy
b) PPS
c) Tubectomy
d) Mini lap
Ans: a
214. Currently married couple, where women is in the reproductive age
a) Potential couple
b) High risk couple
c) Eligible couple
d) Able couple
Ans: c
215. Hydrophobia is typical feature of
a) Plague
b) Rabies
c) Malaria
d) Anthrax
Ans:b
216. STD ara also known as
a) Reproductive disorder
b) Sexual disorder
c) Neurosis
d) Venereal diseases
Ans: d
217. The test used to find out syphilis
a) WIDAL test
b) ELISA test
c) VDRL test
d) Shick test
Ans: c
218. The science of health and its prevention is
a) Biology
b) Sociology
c) Nutrition
d) Hygiene
Ans: d
219. Superv9ised community based treatment is
a) Chemotherapy
b) MPO
c) DOTS
d) Surveillance
Ans: c
220. World AIDS days is
a) April 1st
b) May 1st
c) December 1sdt
d) July 1st
Ans:c
221. Total weight gain during pregnancy
a) 5kg
b) 10kg
a) 15kg
b) 20kg
Ans: b
222. Worm that is transmitted through meat is
a) hookworm
b) tapeworm
c) round worm
d) pinworm
Ans: b
223. Basic unit of society is
a) individual
b) Panchayat
c) family
d) block
Ans: c
224. The panchayat raj institution at block level
a) Grama panchayat
b) Zila poanchayat
c) Grama shaba
d) Panchayat samiti
Ans: d
225. Chemical substances used for purifying water
a) lime
b) KMNO4
c) Bleaching powder
d) Formaldehyde
Ans: c
226. Tuberculosis test is commonly known as
a) Shick test
b) Mantoux test
c) VDRL
d) ELISA
Ans: b
227. The prime source of all water is
a) Sea
b) River
c) Rain
d) Well
Ans: b
228. First step in control of communicable disease is
a) Prevention
b) Surveillance
c) Treatment
d) Notification
Ans: b
229. Method used to diagnose community health problem is
a) Survey
b) Health education
c) Camp
d) lab test
Ans: a
230. Total number of multipurpose health worker in sub centre is
a) 1
b) 2
c) 3
d) 4
Ans:a
231. One sub centre for population of
a) 3000
b) 5000
c) 1000
d) 10000
Ans: b
232. The daily requirement of foilc acid for adult is
a) 50 microgram
b) 300 microgram
c) 200 microgram
d) 100microgram
Ans: d
233. Back bone of publich health nurse is
a) Home visit
b) Records
c) Health education
d) Anganwadi
Ans: a
234. Causative agent for diphtheria
a) Mycobacterium
b) Bordetella
c) Corynebacterium diphtheria
d) Salmpnella
Ans: c
235. Which id didactic method of health teaching
a) Symposium
b) Workshop
c) Group discussion
d) Lecture
Ans:d
236. Panchayat rai insitiution at rural level is
a) Gram sabha
b) Grama panchayat
c) Panchayat samiti
d) Zilla panchayat
Ans: b
237. Most sensitive index of health and level of living of people
a) Crude death rate
b) Birth rate
c) IMR
d) MMR
Ans: c
238. Causes of temporary hardness of water
a) Carbonates
b) Sulphur
c) Chlorides
d) Calcium
Ans:a
239. Characteristics smile in tetanus patient is
a) Opisthotons
b) Ataxia
c) Akinesia
d) Risus sardonicus
Ans: d
241. Poor mans meat is
a) 240. milk
b) Cereals
c) Pulses
d) Fish
Ans: c
242. shick test is used to diagnose
a) TB
b) Diphtheria
c) Vital hepatitis
d) Typhoid fever
Ans:b
243. Dr. salk discovered a vaccine for prevention of
a) Measles
b) Poliomyelitis
c) Small pox
d) Tetanus
Ans: b
244. A substances that suppresses or neutralize bad odours is called
a) Deodorant
b) Antiseptic
c) Disinfection
d) Antibiotic
Ans: a
245. Rabies is a fatal viral disease of
a) Respiratory system
b) Muscular system
c) CNS
d) CVS
Ans: c
246. Test used to detect typhoid
a) WIDAL test
b) Millard test
c) Kahn test
d) Rorschach test
Ans:a
247. Under ICDS program there is Anganwadi worker for
a) 1000 population
b) 20000 population
c) 2000 population
d) 3000 population
Ans: a
248. Very effective method of health teaching
a) Symposium
b) Group discussion
c) Lecture
d) Films and charts
Ans: d
249. Ovulation method of family planning is called as
a) Calendar method
b) Billing method
c) Symptothermal methods
d) Spacing method
Ans: c
250. Health survey and planning committee is
a) Mudhiliyar committee
b) Bhore committee
c) Kartar singh committee
d) Shrisvastav committee
Ans: a
251. An international agency that assists in programmes that benefits child health
a) UNO
b) UNICEF
c) WHO
d) Red cross
Ans: b
252. Oral thrush is caused by
a) Bacteria
b) Fungus
c) Protozoa
d) Virus
Ans: b
253. Occupational disease caused by coal dust is
a) anthracosis
a) Silicosis
b) Asbestosis
c) Bagossosis
Ans: a
254. a safe simple and reliable for of male contraception is
a) Use of condom
b) Vasectomy
c) Coitus interrupts
d) All of the above
Ans: a

256. The best and useful method of refuse disposal in rural area is
a) Dumping
b) Composting
c) Incineration
d) Controlled tipping
Ans: b
257. Sources of vital statistics are
a) National sample survey
b) Census
c) Report form difference institution
d) All of these
Ans: d
258. WHO day is
a) April 7th
b) May 1st
c) December 1st
d) July 1st
Ans:a
259. The best time for insertion of IUD is
a) 40 days after delivery
b) 6 weeks after abortion
c) Intermenstrural period
d) all of these
Ans: d
260. Which of the following disease can transmit though water?
a) typhoid
b) tuberculosis
c) measles
d) chicken pox
Ans: a
261. Birth should be registered with
a) 45 days
b) 14 days
c) 21 days
d) 7 days
Ans:b
262. The causes of MMR is
a) Haemorrhage
b) Sepsis
c) Anaemia
d) All of these
Ans:d
263. Which of the following is water born disease
a) Filariasis
b) Dengue fever
c) Poliomyelitis
d) Influenza
Ans: b
264. Pap smear examination is done to diagnose
a) ca breast
b) ca cervix
c) ca bladder
d) ca rectum
Ans: b
265. Anaemia prophylaxis in pregnancy starts at the
a) 1st month
b) 2nd month
c) 3rd month
d) 4th month
Ans: d
266. The most frequent side effect associated with the use of IUCD is
a) Ectopic pregnancy
b) Expulsion of IUD
c) Rupture of uterus
d) Excessive menstrual flow
Ans:d
267. The most appropriate time to collect sputum specimen form a patient is
a) Before meals
b) After meals
c) On awakening
d) After activity
Ans:c
268. The most important complication of mumps in post pubertal males is
a) Hypopituitarism
b) Sterility
c) Decrease in libido
d) Decrease in androgens
Ans: b
269. The most important aspects of hand washing is
a) Time
b) Soap
c) Water
d) Friction
Ans: d
270. Mode of transmission of pertusis is
a) vector
b) Water
c) Droplet
d) Feco-oral
Ans: c
271. time required to measure the Monteux test on anterior left arm is
a) 24-48 hours
b) 48-72 hours
c) 10-12 hours
d) 2-4 hours
Ans: c
272. Media of transmission of bovine type of tubercle bacilli is
a) Milk
b) Water
c) Air
d) Food
Ans: a
273. A Fungal infection is
a) Malaria
b) Athlets foot
c) Filarias
d) Typhus fever
Ans: b
274. Number of times an healthy person can donate blood in a year
a) 3-4 times
b) 4-6 times
c) 7-8 time
d) 5 -6 times
Ans: a
275. The following are the content of ORS except
a) Sodium chloride
b) Potassium chloride
c) Sugar
d) Calcium carbonate
Ans:d
276. The course of oral pills should be started on the – day of menstrual cycle
1st
a) 2nd
b) 7th
c) 5th
d) 10th
Ans: c
277. A permanent method of sterilization is
a) MTp
b) Diaphragm
c) Tubectomy
d) D and C
Ans: c
278. Information given by a suffered about is illness is
a) Sign
b) Symptoms
c) Observation
d) diagnosis
Ans: b
279. Several people in a town got the attack of leukemia
A possible reason may be
a) exposed to radiation
b) drinking polluted water
c) smoking
d) breathing impure air
Ans: a
280. any waste which is generated during the diagnosis treatment or research in the
hospital is called
a) biomedical waste
b) infected waste
c) solid waste
d) pharmaceutical waste
Ans:a
281. Complication of chikungunya includes the following except
a) encephalomyelitis
b) fulminant hepatitis
c) erosive arthritis
d) carditis
Ans: d
282. CNS manifestation chikungunya includes the following
a) photopobia
b) gastritis
c) joint pain
d) myalgia
Ans:a
283. Chikungunya is caused by
a) arbovirus
b) aedes aegypti
c) cules
d) anopheles
Ans: b
284. drugs which cross the placental barrier
a) bioactive drugs
b) teratogenic drugs
c) genotoxic drug
d) none of these
Ans: b
285. a direct mode of transmission
a) droplet
b) fomite
c) vector
d) vehicle
Ans:a
286. physician induced infection is
a) opportunistic infection
b) nosocomial infection
c) iatrogenic infection
d) indirect infection
Ans: c
287. first case of common disease in a population
a) index case
b) primary care
c) carrier
d) all of these
Ans:b
288. first case come to the attention of the investigator is
a) index case
b) primary care
c) carrier
d) all of these
Ans:a
289. contraindication for IUD insertion
a) pyrexia
b) multipara
c) irregular period
d) nullipara
Ans:d
290. calendar method is otherwise known as
a) abstinence
b) symptothermal method\
c) safe period method
d) withdrawal method
Ans: c
291. MTP act permit abortion onlyu up to
a) 20 weeks
b) 24 wweeks
c) 28 weeks
d) 30 weeks
Ans:a
292. An injectable contraceptive
a) Mala-D
b) Mala-L
c) NET-EN
d) Cu-t
Ans: c
293. Male pill
a) Quinestrol
b) Grosspol
c) Norethisterone
d) NET-En
Ans: b
294. 3rd generation IUD’s are
a) Copper impregnated
b) Hormonal
c) Temporary
d) Nylon
Ans: b
295. The tile of IUD are made up of
a) Nylon
b) Silicon
c) Silastic
d) Copper
Ans: a
296. The indicator of prevalence of contraceptive practices in the country is
a) Eligible couple register
b) Couple protection rate
c) Net reproduction rate
Ans:b
297. The basic document of family planning is
a) Eligible couple register
b) Couple protection rate
c) Net reproduction rate
Ans:a
298. The first step in water purification is
a) Sedimentation
b) filtration
c) storage
d) Chlorination
Ans: c
299. Very effective method of communication is
a) seminar
b) Symposium
c) Group discussion
d) Lecture
Ans:c
300. Waste water from community which does not contaij human excreta is
a) Sewage
b) Contaminated water
c) Infected wate
d) Sullage

Ans: d
301. Anganwadi cneter is tha part of
a) ICDS
b) PHC
c) CHC
d) Voluntary agency
Ans: a
302. Most commonm occupational hazards of computer worker is:
a) cataract
b) Lung disease
c) CarpEl tunnel syndrome
d) Cancer
Ans: C
303. Community health nurse should be
A) Targer oriented
B) Population oriented
C) Disease oriented
D) Age and sex specific
Ans: b
304. Which of the following is included in helath sector in India?
a) Water supplu and sanitation
b) Control of communicable diseases
c) Medical education and research
d) All of the above
Ans:d
305. IMNCI stand for
a) Integrated management of neonatal and childhood illness
b) International management of neonatal and childhood illness
c) International morbidity of neonatal and childhood illness
d) Intercordinated management of neonatal and childhood illness
Ans:a
306. How many essential componenets of PHC has been out lined by Alma Ata
decrlaration
a) 4
b) 10
c) 8
d) 6
Ans:c
307. The female helath worker can detect all oif the disease except?
a) anaemia
b) renal disease
c) malpresentation
d) Hydroamniosis
Ans: b
308. Sickness is astate of
a) Social disaffection
b) Subjective state of a person feeling unwell
c) Impaired [physiological function
d) Impaired psychological function
Ans: c
309. Morbidity in a community can be estimated by
a) Active surveillance
b) Passive surveillance
c) Sentinel surveillance
d) Monitoring surveillance
Ans: b
310. ASHA stand for
a) Accredited social health activist
b) Accredited section health activist
c) All social health activities
d) Accredited state health activist
Ans:a
311. All of the above is method of health promotion except
a) Nutritional education
b) Immunization
c) Behavioural changes
d) Healthful housing
Ans: b
312. The concept of primary health centr was introduced in to Indian health system on
the report of
a) Bhore committee
b) Mudhilayar committee
c) Chadha committee
d) Shrivastva committee
Ans: a

313. Per capita space of a student in a class room not less than
a) 20 sq ft
b) 10 sq ft
c) 30 sq ft
d) 40 sq ft
Ans: b

314. Syndromic approach of STD management is applicable at


a) PHC level
b) CHC level
c) District level
d) None of the above
Ans:a

315. PHC serves a population of


a) 30000-20000
b) 40000-30000
c) 50000-40000
d) None of these
Ans:a

316. The depth of bore hole latrine varies from


a) 6-8 m
b) 10-12 m
c) 4-8m
d) 8-12 m
Ans:c

317. One block consist of


a) 100 villages
b) 200 vilages
c) 300 villages
d) 400 villages
Ans: a

318. Janani suraksha yojna aijmed to reduce


a) MMR
b) Neonatal mortality rate
c) Underfive mortality rate
Ans:a and b
Ans:d
319. The incubation period of pertuisis is
a) 1-7 days
b) 7-14 days
c) 1-4 days
d) 8-12 days
Ans: b
320. a patient with spoutum positive pulmonaty TB is on anti TB therapy for last 5
months but the patient is still positive for acid fast bacilli, this case will be termed as
a) New case
b) Default case
c) Failure case
d) Relapse case
Ans:b

321. Which of the following test is reliable to screen Dm in the community


a) Urine examination for glycosuria
b) FBS
c) RBS
d) GTT ( glucose tolerance test)
Ans: d
322. Definition of blindness given by WHO includes
a) visual acuity <3/60
b) avisual acuity <4/60
c) visual acuity <5/60
d) visual acuity </60
Ans:a
323. Oral rehydration therapy programme is stated tin
a) 1967-68
b) 1957-58
c) 1996-97
d) 1985-86
Ans: d
224. Still birth rate included death of foetus weighing
a) 5oogm
b) 800gm
c) 1000gm
d) 900gm
Ans: c
325. ESI was stated in
a) 1952
b) 1940
c) 1948
d) 1846
Ans:c
326. CSSM programme was launched in
a) 1992
b) 1948
c) 1999
d) 1956
Ans:a
327. RCH programme was launched in
a) 1997
b) 1987
c) 1948
d) 1957
Ans: a
328. in 1987 which programme was launches
a) National blinness control programme
b) National AIDS control programme
c) Leprosy control program
d) Malaria control program
Ans: b
329. Period of training for local dais is
a) 3 weeks
b) 30days
c) 200 hours
d) 1 wek
Ans: b
330. Duration of train9ing for village health guide is
a) 200 hours
b) 400 hours
c) 1 week
d) 30 days
Ans: a
331. Indian red cross society was established in
a) 1920
b) 1950
c) 1930
d) 1940
Ans: a

332. Carlina is 15 months old and weighs 5.5 kgs and it is her initial visit. Her mother
says that Carlina is not eating well and unable to breastfeed, he has no vomiting, has no
convulsion and not abnormally sleepy or difficult to awaken. Her temperature is 38.9
deg C) Using the IMCI strategy, if you were the nurse in charge of Carlina, how will
you classify her illness?
a) a child at a general danger sign
b) very severe febrile disease
c) severe pneumonia
d) severe malnutrition
Ans: b
333. . Shelly has had diarrhea for 8 days. There is no blood in the stool, he is irritable,
his eyes are sunken, the nurse offers fluid to Shelly and he drinks eagerly. When the
nurse pinched the abdomen it goes back slowly. How will you classify Shelly’s Illness?
a) Moderate dehydration
b) Some dehydration
c) Severe dehydration
d) No dehydration
Ans: b
334. To determine the possible sources of sexually transmitted infections, which is the
BEST method that may be undertaken by the public health nurse?
A) Contact tracing
B) Community survey
C) Mass screening tests
D) Interview suspects
Ans:a
335. . Antiretroviral agents, such as AZT are used in the management of AIDS. Which
of the following is not an action expected of these drugs?
A) They prolong the life of the client with AIDS
B) They reduce the risk of opportunistic infections
C) They shorten the period of communicability of the disease
D) They are able to bring about a cure of the disease condition
Ans:d
336. A village had an outbreak of German measles. To prevent congenital rubella, what
is the BEST advice that you can give to women in the first trimester of pregnancy in the
village?
a) Advice them on the sign of German Measles
b) Avoid crowded places, such as markets and moviehouses
c) Consult at the health center where rubella vaccine may be given
d) Consult a physician who may give them rubella immunoglobulin
Ans:d
337. . A 4-year old client was brought to the health center with chief complaint of severe
diarrhea and the passage of “rice water”. The client is most probably suffering from
which condition?
A) Giardiasis
B) Cholera
C) Amebiasis
D) Dysentery
Ans:b
338. You are the PHN in the city health center. A client underwent screening for AIDS
using ELISA) His result was positive. What is the best course of action that you may
take?
A) Get a thorough history of the client, focusing on the practice of high risk behavior
B) Ask the client to be accompanied by a significant person before revealing the result.
C) Refer the client to the physician since he is the best person to reveal the result to the client
D) Refer the client for a supplementary test, such as Western blot, since the ELISA result
maybe false
Ans:d

339. . A 32 year old client came for consultation at the health center with the chief
complaint of fever for a week. Accompanying symptoms were muscle pains and body
malaise. A week after the start of fever, the client noted yellowish discoloration of his
sclera. History showed that he waded in flood waters about 2 weeks before the onset of
symptoms. Based on this history/ which disease condition will you suspect?
A) Hepatitis A
B) Hepatitis B
C) Tetanus
D) Leptospirosis
Ans:d

340. A 2-month old infant was brought to the health center for immunization. During
assessment, the infant’s temperature registered at 38.1 C) Which is the best course of
action that you will take?
A) Go on with the infants immunization
B) Give paracetamol and wait for his fever to subside
C) Refer the infant to the physician for further assessment
D) Advise the infant’s mother to bring him back for immunization when he is well
Ans:a
341. . A pregnant woman had just received her 4th dose of tetanus toxoid)
Subsequently, her baby will have protection against tetanus for how long?
A) 1 year
B) 3 years
C) 10 years
D) Lifetime
Ans:a
342. A 4-month old infant was brought to the health center of cough. Her respiratory
rate is 42/minute. Using the IMCI guidelines of assessment, her breathing is considered;
A) Fast
B) Slow
C) Normal
D) Insignificant
Ans:c
343. A 5-month old infant was brought by his mother to the health center because of
diarrhea occurring 4 to 5 times a day. His skin goes back slowly after a skin pinch and
his eyes are sunken. Using the IMCI guidelines, you will classify this infant in which
category?
A) No signs of dehydration
B) Some dehydration
C) Severe dehydration
D) The data is insufficient
Ans:b
344. . Based on the assessment, you classified a 3-month old infant with the chief
complaint of diarrhea in the category of SOME DEHYDRATION. Based on the IMCI
management guidelines, which of the following will you do?
A) Bring the infant to the nearest facility where IV fluids can be given
B) Supervise the mother in giving 200 to 400 ml of Oresol in 4 hours
C) Give the infant’s mother instructions on home management
D) Keep the infant in your health center for close observation
Ans:b
345. A mother is using Oresol’ in the management of diarrhea of her 3-year old child)
She asked you what to do if her child vomits. You will tell her to:
A) Bring the child to the nearest hospital for further assessment
B) Bring the child to the health center for IV therapy
C) Bring the child to the health center for assessment by the physician
D) Let the child rest for 10 minutes then continue giving Oresol more slowly
Ans:d
346. A 1 1/2 year old child was classified as having 3rd degree of protein energy
malnutrition, kwashjorkor. Which of the following signs will be most apparent in this
child?
A) Voracious appetite
B) Wasting
C) Apathy
D) Edema
Ans:d
347. Assessment of a 2-year old child revealed “baggy pants”. Using the IMCI
guidelines, how will you manage this child?
A) Refer the child urgently to a hospital for confinement
B) Coordinate with the social worker to enroll the child in a feeding program
C) Make a teaching plan for the mother, focusing on the menu planning for her child
D) Assess and treat the child for health problems like infections and intestinal parasitism
Ans:a
348. . A mother brought her daughter, 4 years old, to the RHU because of cough and
colds. Following the IMCI assessment guide, which of the following is a danger sign that
indicates the need for urgent referral to a hospital?
A) Inability to drink
B) High grade fever
C) Signs of severe dehydration
D) Cough for more than 30 days
Ans:a
349. A mother brought her 10 month old infant for consultation because of fever which
started 4 days prior to consultation. To determine malaria risk, what will you do?
A) Do a tourniquet test
B) Ask where the family resides
C) Get a specimen for blood smear
D) Ask if the fever is present everyday
Ans:b
350. The following are strategies implemented by the MOH to prevent mosquito-borne
diseases. Which of these is most effective in the control of Dengue fever?
A) Stream seeding with larva-eating fish
B) Destroying breeding places of mosquitoes
C) Chemoprophylaxis of non-immune persons going to endemic areas
D) Teaching people in endemic areas to use chemically treated mosquito nets
Ans:b
351. A mother brought her 10 month old infant for consultation because of fever which
started 4 days prior to consultation. To determine malaria risk, what will you do?
A) Do a tourniquet test
B) Ask where the family resides
C) Get a specimen for blood smear
D) Ask if the fever is present everyday
Ans:b
352. The following are strategies implemented by the MOH to prevent mosquito-borne
diseases. Which of these is most effective in the control of Dengue fever?
A) Stream seeding with larva-eating fish
B) Destroying breeding places of mosquitoes
C) Chemoprophylaxis of non-immune persons going to endemic areas
D) Teaching people in endemic areas to use chemically treated mosquito nets
Ans:b
353. A 3-year old child was brought by his mother to the health center because of fever
of 4-day duration. The child had a positive tourniquet test result. In the absence of other
signs, which of the most appropriate measure that the PHN may carry out to prevent
Dengue shock syndrome?
A) Insert an NGT and give fluids per NGT
B) Instruct the mother to give the child ORS
C) Start the patient on IV Stat
D) Refer the client to the physician for appropriate management
Ans:b

354. Being a community health nurse, you have the responsibility of participating in
protecting the health of people. Consider this situation: Vendors selling bread with their
bare hands. They receive money with these hands. You do not see them washing their
hands. What should you say/do?
a) “Miss, may I get the bread myself because you have not washed your hands”
b) All of these
c) “Miss, it is better to use a pick up forceps/ bread tong”
d) “Miss, your hands are dirty. Wash your hands first before getting the bread”

Ans: c
355. Parents who want to have their child immunized against smallpox approach a
community health nurse. What is the best response by the nurse to these parents?
a) "I'll see what I can do to get your child the immunization."
b) "You should be concerned that your child has their scheduled immunizations."
c) "You'll have to ask your doctor to order this for you."
d) "Smallpox disease has been eliminated from the world and could only be re-
introduced by bioterrorism."
Ans:d
356. A community health nurse is providing information on Post-polio syndrome (PPS).
Of the following clients, who would most benefit from this information? Clients who:
a) Parents of children who are receiving the immunization.
b) Have been immunized against the disease.
c) Had the disease as a child.
d) Are immigrating into the U.S. from other countries.
Ans: c
357. Some communicable diseases are spread by airborne transmission. Of the following
list, select those that apply.
a) Meningococcal meningitis
b) Hepatitis A
c) Influenza
d) Hepatitis B
e) Chicken pox
Ans:e
359. For several months, a community health nurse sees a client who was exposed to
viral hepatitis. The nurse suspects the client is in the prodromal period of the disease
because the client is experiencing the following symptoms except :
a) Nausea.
b) Has a yellowish coloration to their skin.
c) Malaise.
d) Fatigue.
Ans:b
360. A community health nurse is working in a walk-in clinic that serves a large number
of HIV/AIDS clients. Understanding the psychological consequences of this chronic
disease, what would be an appropriate assessment question for the caregivers of these
clients?
a) Have you noticed anything different about the client's behavior lately?"
b) "Does the client seem depressed?"
c) "Other than the physical symptoms you reported last visit, have there been any new
problems that have developed?"
d) "Do you attend a support group?"
Ans:a

361. An infection control nurse in a hospital is looking at trends and sick days used in
each department and identifies the medical records department as being a high use area
for sick days. Understanding the influence of the physical environment, the nurse
should:
a) Recommend employees attend a stress-reduction seminar.
b) Recommend strategies on how the employees can boost their own immune systems.
c) Recommend that employees be screened for health problems before they would be
hired to this area.
d) Assist the department in developing a cleaning schedule for their keyboards and
phones.
Ans:d
362. A school nurse is providing immunization information at a parent's night. After the
presentation, a parent approaches the nurse and relates that even though their child has
received all immunizations recommended, the parent, because of variables out of their
control, was never immunized against polio. The nurse should
a) Notify the public health department and the office of social services.
b) Tell the parent that because of their age, they are no longer at risk to develop the
disease.
c) Recommend that the parent receive IPV.
d) Explain the aspect of "herd immunity" to the parent.
Ans:c
364. A nurse employee of a hospital has decided to not receive the influenza vaccine and
states that it's because they never get ill and prefer not to risk a reaction to the
immunization. The infection control nurse should respond:
a) The shot is free to employees, so you should get the immunization."
b) "Why would you feel that getting immunized is not in your best interest?"
c) "You can't get sick from the immunization."
d) "The vaccine doesn't only protect you, but your clients and the hospital as well."
Ans:d
365. A pregnancy client with HIV is being seen in the community health clinic. The
nurse, understanding vertical transmission, makes this statement to the client:
a) "We will need to talk about birth control options so you don't become pregnant
again."
b) "Maybe your baby's virus won't develop into AIDS."
c) "We will start treating your baby for HIV as soon as it's born."
d) Your baby's risk of acquiring HIV from you will be much less if you are receiving
treatment for the infection."
Ans:d

366. A client visits the community health office. During the initial interview, the nurse
learns that the client is not working, even though she is only 45-years old, and states
that "it's just too difficult" to get anything done. The client also states that she
frequently has difficulty with pain, especially when the seasons change. Understanding
the incidence of chronic disease, the nurse should ask the client about which of the
following?
a) Arthritis
b) Chronic Obstructive Pulmonary Disease
c) Asthma
d) Heart disease
Ans: a
367. A community health nurse is conducting a hypertension-screening clinic. Because
of the nurse's understanding of chronic disease, the screen should test for:
a) Diabetes.
b) Heart disease.
c) Stress.
d) Chronic Obstructive Pulmonary Disease
Ans: c
368. A public health nurse is aware that a particular community has a high incidence of
several chronic diseases including hypertension and circulatory problems, as well as
high rates of coronary artery bypass surgeries. Aside from genetic and stress-related
factors, which environmental element should the nurse investigate?
a) Structures
b) Air quality
c) Drinking water
d) Soil and ground
Ans: c

369. During a "Women's Day" health fair, a community health nurse addresses the
incidence of breast cancer and screening activities. Which group of women may not be
aware of their heightened risk for breast cancer? Women who:
a) Have a family history of breast cancer.
b) Work nights.
c) Women who eat low-fat diets.
d) Are over age 50.
Ans: b
370. A public health nurse is working for Indian Health Services (IHS). Understanding
chronic health problems, particularly mortality rate associated with cancer in Native
Americans, the nurse should advocate for which type of community service?
a) Nutritional services
b) Transportation
c) Respite care
d) Health care services
Ans: b
371. A community health nurse is working to help improve the sociocultural health
dimension in a particular neighborhood. Select the following activities the nurse could
help implement that would be examples of sociocultural factors affecting the
development of chronic disease:
a) Help parents understand specific behaviors associated with increased risk for obesity,
such as a child's sedentary lifestyle.
b) Implementing a non-smoking campaign in a local middle school.
c) Teach an exercise class for after-school programs to implement.
d) Educate providers of school lunch programs regarding portion control and appropriate
meal size
Ans: a
372. A community health nurse has been working with a local school system in its
efforts to control tobacco use by their students. An example of a health promotion
activity for this situation would be:
a) Screening students for asthma or other respiratory problems.
b) Educating instructors on how to recognize students who may be using tobacco.
c) To promote compliance by teaching a smoking cessation class about the effects of
tobacco.
d) Helping to enforce penalties on store owners that sell tobacco to minors.
Ans: d

373. A public health nurse is aware of an increased number of diabetes cases in a


particular area of a city that is mainly inhabited by poor, unemployed citizens who do
not have insurance. A secondary prevention measure aimed at the population level that
would promote early treatment would be:
a) Helping newly diagnosed clients how to prevent further problems of their disease.
b) Surveying this particular group regarding information about dietary preferences,
exercise, and family history.
c) Educating teachers in the local school system how to identify signs and symptoms of
diabetes.
d) Assisting a local hospital with funding to provide for free, monthly, random blood
sugar testing.

Ans: d
374. A community health nurse is assisting a client with chronic pain in achieving an
acceptable level of pain management. The client has done much research on pain
management techniques and has kept a journal outlining details of techniques that have
helped in the past, as well as those that did not help with their symptoms. The nurse has
found several community resources that the client was able to utilize as well. Which
model of self-management does this represent?
a) Economic model
b) Collaborative model
c) Medical model
d) Self-agency model
Ans: d
375. A nurse educator is teaching the principal elements of several common
psychological conditions. Of the following, select those that are representative of anxiety
disorders:
a) Bipolar disorder.
b) Panic disorder.
c) Post-traumatic stress disorder.
d) Phobia.
e) Obsessive-compulsive disorder.

Ans:b
376. A community health nurse is working with clients who are diagnosed with
schizophrenia. All the clients are within a family structure and have been utilizing the
support system suggestions and other interventions put in place by the nurse. When
formulating a diagnosis for the family, the nurse selects which of the following?
a) Family coping; potential for growth related to utilization of supportive services.
b) Individual defensive coping related to psychosocial stressors.
c) Caregiver role strain related to insufficient available support.
d) Ineffective family coping related to unmet psychosocial needs.
Ans:a

377. A community health nurse is implementing health education classes and focuses
one of the presentations on depression. Of the following clients, which is at the highest
risk for developing depression?
a) A middle-aged client who also has diabetes.
b) A teenager with asthma.
c) An adolescent client whose parents are divorced.
d) An elderly client who is homebound.
Ans: d
378. A community health nurse is working with post-trauma victims of a Category 5
hurricane. Of the following activities, which would be most appropriate in promoting
resilience among this group?

a) Helping the community rebuild their homes.


b) Assessing for signs of depression.
c) Building the group's coping skills by allowing them to create their own support group
within the community.
d) Inviting speakers to discuss the after-effects of the trauma.

Ans: c
379. A family is experiencing severe stress after their baby was born with developmental
and physical birth defects. The best intervention to prevent family dysfunction would be
to:
a) Refer the family to counseling services.
b) Provide information to the family regarding placement of the child in an institution.
c) Provide a list of services available for respite care.
d) Assist the family to cope with the situation.
Ans: a

380. A community health nurse is working with a hospital emergency department (ED)
so treatment of the homeless clients who are chronically mentally ill can occur. Together
with the hospital, the ED implements which of the following to represent a population
level service that meets this need?
a) A weekly screening clinic in the ED.
b) One-on-one visitation with the clients and community health nurse to promote a
trusting relationship.
c) A walk-in, mental health clinic as part of the ED.
d) Referring each client to an appropriate provider who specializes in mental health
problems and is willing to take clients who can't pay
Ans:c

381. A client has been taking Nardil, a monoamine oxidase (MAO) inhibitor, and comes
to the community health office complaining of having headaches and feeling dizzy. After
initial assessment, the nurse should ask which question?
a) "When did you last see your primary care provider?"
b) "Have you been feeling suicidal?"
c) "Do you think this medication is making you sick?"
d) "What type of foods have you been eating?"

Ans:d
382. A client that the community health nurse has been working with was placed on
Prozac (Fluoxetine) for depression. The client comes in for follow-up after being on the
medication one week and states that they are still depressed and haven't felt any better.
The nurse should respond with which of the following?
a) "Are you sure you're taking the correct dose?"
b) "You probably need to be on a larger dose for this medication to help you."
c) "Don't be discouraged, it sometimes takes three weeks for this medication to help you
feel any better."
d) "Maybe there's a better medication that would help you."
Ans:c

383. A community health nurse has made available a list of trained volunteers to assist
families with members who have chronic mental health problems, including dementia
and Alzheimer's disease. These volunteers are available to provide the primary care
givers opportunities for respite services. This is an example of what type of care for
clients with mental health problems?
a) Prevention
b) Maintenance
c) Treatment
d) Secondary
Ans:b

384. A school nurse is taking a survey of high school students and their out-of-school
activities. Of the following, which student is most at risk to abuse psychoactive
substances? The student who:
a) Has a history of drug abuse in the family.
b) States they like the way they felt after one experimental episode.
c) Is a high achiever and frequently feels stress from having too much schoolwork.
d) Is a loner and doesn't have many friends.
Ans:b

385. A school nurse working in a junior high school setting is always alert for students
who may be abusing drugs. Which of the following is the most commonly abused drug
in this age group?
a) Heroin
b) Cocaine
c) Alcohol
d) Sedatives
Ans:c

386. A school nurse is working with a high school student suspected of having anorexia.
During the initial interview, the nurse should ask the student this question:
a) "What do you know about nutrition?"
b) "What do other people in your family look like?"
c) "Does either of your parents abuse alcohol?"
d) "How much do you weigh?"

Ans:c
387. A public health nurse is working in a shelter frequented by clients who are alcohol
abusers. Because of the physiological systemic effects, the nurse should also screen for:
a) Suicide.
b) Respiratory problems.
c) Depression.
d) Musculoskeletal problems

Ans:b
389. A client of a clinic is a substance abuser. Once the nurse has identified this being
the case, the nurse should
a) Educate the client regarding the effects of substance abuse.
b) Refer the client to a support group.
c) Refer the client for in-patient substance withdrawal.
d) Assess the client's background for causality of the abuse

Ans:d
390. A nurse is working in a racially diverse area of the community. While
understanding the cultural tendencies of each group, the nurse knows which ethnic
group is least likely to abuse alcohol?
a) East Indians
b) Native Americans
c) Caucasian males
d) African-Americans
Ans: d

382. A public health nurse is working with a client who was an alcohol abuser
throughout her pregnancy. Of the following, which activity would be most appropriate
throughout the next years for the child, even though at birth, there were no adverse
effects of the substance abuse?
a) Developmental assessment
b) Screening for liver disease
c) Physical assessment
d) Since the baby had no problems at birth, no follow-up would be necessary
Ans: a
393. A parish nurse works closely with the church's youth group. Of the following
adolescents, which is least likely to fall risk for substance abuse? The child who:
a) Is not involved in church functions.
b) Has above-average grades in school.
c) Has no family history of substance abuse.
d) Has a mother involved in anti-drug campaigns.

Ans:d
394. A geriatric nurse specialist has been visiting a senior community center and would
like to screen the senior clients who utilize the center for substance abuse. The nurse
realizes this is an important intervention in the care of the elderly client because:
a) Substance abuse is not a problem in this age group.
b) There are several community resources available for the elderly client who is a
substance abuser and the sooner it is detected, the better for the client.
c) Most primary care providers for the elderly are able to handle this type of care in their
clinics.
d) It is often overlooked and under treated
Ans:d

395. A public health nurse is a pandemic specialist working for the CDC and is tracking
avian bird flu, which has been identified in three countries. The nurse identifies this as
which phase of a pandemic:
a) hase four.
b) Phase three.
c) Phase two.
d) Phase one
Ans: d

396. A public health nurse is a specialist in disaster planning. After a major subway
collapse, the nurse specialist is contacted for support. The nurse begins to assess the
damage and identifies this as a(n):
a) Intentional disaster.
b) Man-made disaster.
c) Natural disaster.
d) Structural disaster.
Ans: b
397. A public health nurse is researching the internal health system of a Middle-Eastern
country and how the current government continues to terrorize its citizens.
Understanding the definition of terrorism, the four elements that characterize terrorism
are: Select those that apply.
a) Political motivation
b) Use of extreme violence
c) Persuasion with fear
d) Premeditation in the use of violence
e) Personal motivation
Ans: except A all are right

398. A public health nurse is involved in disaster planning at all stages of response. The
nurse has surveyed the community and determines that there are a large number of
elderly clients in a particular area that would have to be evacuated during a flood. This
is an example of:
a) Vulnerability assessment.
b) Capability inventory.
c) Soft mitigation.
d) Hard mitigation.

Ans: a
399. A group of nursing students volunteered to travel to an area of the state just struck
by a hurricane to provide care and services where needed. These students would be
assisting at which stage of the disaster?
a) Restoration stage
b) Emergency stage
c) Impact stage
d) Reconstruction stage

Ans: b

400. During a winter ice storm in an upper Midwestern state, 75% of the electrical lines
were downed, leaving a large portion of the population without electricity. Many of
these people were living in rural areas. The National Guard was called to assist with
emergency efforts and act as first responders in the event of a medical emergency since
most of the roads had not yet been cleared of ice and snow. These soldiers would be
considered which level of victim in this case?
a) Third level
b) Primary level
c) Secondary victims
d) Fourth level
Ans: a

401. A community health nurse is working in a state that is frequently hit by tornadoes.
After one such episode, a grade school was hit and several children were killed. At a
subsequent town meeting, the nurse was available to provide information regarding
counseling services and post-trauma symptoms. This is an example of intervention in
which consideration that is assessed in a disaster?
a) Biophysical
b) Physical environmental
c) Psychological
d) Sociocultural

Ans: c
402. A young soldier returning home from duty in a war zone tells the nurse at the state-
side army base that sometimes he felt the media was a bigger enemy than the terrorists
he was fighting. The nurse, understanding the considerations to assess in this situation,
believes this soldier is reacting to:
a) Sociocultural factors.
b) Behavioral factors.
c) Physical environmental factors.
d) Psychological factors.

Ans: d
403. Following a devastating hurricane, community health nurses play a role in disaster
planning. At the mitigation level, a nurse would be involved in which of the following?
a) Helping to develop an evacuation plan for hospitals and health care facilities.
b) Advocating for legislation that enforces inspection of sea walls and dams.
c) Assisting with developing a disaster plan.
d) Identifying temporary shelters for treatment and health care of victims
Ans: b

404. Hidden costs associated with health issues in the workplace include: (Select all that
apply.)
a) Employee turnover.
b) Poor-quality work.
c) Low productivity.
d) Costs related to sickness.
e) Accidents.
Ans: b, c, e

405. An occupational health nurse is working with a local hospital to develop several
different programs for health promotion. The efforts came about as a result of
community survey among factory workers, who identified the absence of health
promotion activities offered in accordance with shift changes at several industrial
plants. This is an example of which level of services in the public health focus?
a) Prevention level
b) Individual
c) Systems
d) Community
Ans: c

406. Among the standards for nursing in the occupational health field, one will find the
code of ethics that includes: (Select all that apply.)
a) Safeguarding employees from unethical or illegal actions.
b) Safeguarding employees rights to privacy.
c) Maintaining personal competence.
d) Protecting the rights of the employers.
Ans: a, b, c
407. An occupational health nurse is reviewing the statistics of the previous month's
injuries among employees. Addressing the needs of all workers, but understanding the
statistics of workplace injuries, the nurse should:
a) Provide suggestions to enhance the creativity of older workers, which will help their
overall safety habits.
b) Focus attention of workplace safety on the older worker.
c) Make sure older workers have a safer environment, as they are more prone to
accidents.
d) Mandate that employees under age 25 attend quarterly safety in-services.

Ans: d
408. A nurse is employed by a gas company, and a large number of the employees are
truck drivers that deliver petroleum to smaller distributorships. Understanding the
risks associated with this particular group, the nurse will be sure to perform annual
screens that focus on:
a) Cognition and memory.
b) Hypertension.
c) Skin problems.
d) Vision.
Ans: a

409. An occupational health nurse working within a hospital setting would serve the
nursing personnel best by providing what type of in-service on a regular basis?
a) Stress reduction
b) Pain management
c) Body mechanics
d) Infection control

Ans: c
410. A nurse working for a large business has noticed after annual health screenings
that among the young adult population, there are a high number of employees with
high blood pressure and high cholesterol. Understanding how psychological factors
affect overall health, the nurse should also screen for:
a) Anxiety disorder.
b) Depression.
c) Schizophrenia.
d) Bi-polar disorder
Ans: b

411. A nurse is providing health care to employees in a large factory. Aside from in-
service on body mechanics and prevention of musculoskeletal injuries, the nurse should
next be sure that employees are:
a) Screened annually for hypertension.
b) Aware of mental health services within the community.
c) Provided ear protection.
d) Provided information about stress reduction

Ans: c

412. A community health nurse is working where there is a group of migrant workers
employed at various hog farms in the surrounding area. The nurse has seen a fair
number of these workers as they come to the office with asthma-like symptoms. The
nurse understands that there is little legislative support for environmental controls since
most of these employers do not have to worry about regulatory agencies such as OSHA
because:
a) State and federal laws regulate these types of agricultural facilities.
b) Regulatory agencies are not involved in agriculture.
c) They employ less than 10 people.
d) The employees do not have insurance.
Ans: c

413. A community health nurse is planning several health education programs and
focuses on the assumption that members of the community will be able to make
informed health decisions. This nurse is utilizing which of the following concepts of
public health nursing?
a) Rule of social structures
b) New public health
c) Preventive medicine
d) Miasma control
Ans: b

414. Having a sound understanding of health promotion, the public health nurse works
to implement programs within the community that provide opportunities for clients to
become involved with and provide input in health policies. This is an example of a(n):

a) Behavioral model of health promotion.


b) Medical model of health promotion.
c) Socioeconomic model of health promotion.
d) Educational model of health promotion.
Ans: c

415. A community health nurse is working with several agencies in planning to send a
group of health care providers on a mission to a third-world country. One of the goals
of the mission is to assist the local health care providers with environmental and
sanitary conditions, while respecting their cultural beliefs and practices. This nurse is
emphasizing which WHO strategy to promote global health?

a) Reorientation of health services to a health promotion focus


b) Advocacy for conditions favorable to health
c) Development of environments that support health
d) Development of healthy public policy
Ans: b

416. A school nurse is working with several students who have a history of abusing
alcohol on a regular basis. Based on interaction with these students, the nurse takes the
role of assuming that these students do not understand the significance of their actions.
Which stage of the Precaution Adoption Process model are these students portraying?

a) Stage 2
b) Stage 4
c) Stage 3
d) Stage 1
Ans: d

417. A home health nurse is working with a client who has physical therapy ordered
three times per week as part of rehabilitation from a stroke. The client's spouse is not
able to get them to the physical therapy department until late in the afternoon, and the
client is unable to access the public transportation system in their neighborhood. The
couple has no children who can help out. Even though the client understands the need
for therapy, he or she cannot get there. This is an example of which component of the
PRECEDE-PROCEED model of health promotion?

a) PROCEED
b) PRECEDE
c) Reinforcing factors
d) Personal factors
Ans: b
418. A school health nurse decides to present a health promotion activity to high school
students. The nurse implements a panel discussion with recent graduates-some who
used alcohol, some who did not-and offers information about alcohol abuse among
teens. This nurse is utilizing which strategy of health education?

a) Positioning strategy
b) Marketing mix
c) Social marketing
d) Promotion
Ans: a

419. A community health nurse is presenting a health education class to a group of


senior citizens regarding exercises to promote cognitive health. As part of the program,
the nurse presents a slide presentation and handouts. Which of the learning domains is
the nurse working through?

a) Affective domain
b) Perceptual domain
c) Cognitive domain
d) Psychomotor domain
Ans: c

420. A nurse educator begins the unit with an entirely new concept to students. In
assessing the health education needs of the students, this educator must first

a) Identify learning goals.


b) Find out what the students understand about the concept.
c) Develop objectives.
d) Prioritize learning needs.
Ans: b

421. A home health client tells the nurse he has been researching his newest medication
on the Internet. In assisting this client and evaluating the information they received,
which of the following is the best response by the nurse?

a) "Show me the material you found."


b) "I think your doctor should explain any questions you might have."
c) "It's better to receive this type of information from your pharmacist.
d) "Information from the Internet is usually not reliable."
Ans: a

422. A home health nurse is working with a client who lives in a rural setting and
requires multiple outside services for rehab. So far, the nurse has made referrals for
occupational therapy, speech therapy, physical therapy, and the state job retraining
services. This is an example of which model of case management?

a) Full-service model
b) Clinical model
c) Hybrid model
d) Broker model
Ans: d

423. A client who has utilized the community health system numerous times has
returned, needing a lengthy variety of health care services. The nurse working with this
client is also aware the client has a history of non-compliance and poor follow-through.
The case management model that would best serve this client is the:
a) Systems model.
b) Full-service model.
c) Broker model.
d) Hybrid model.
Ans: b

424. A client with multiple, chronic health problems is scheduled for discharge. The
client has been difficult to work with at times and has a limited support system. The
client will require extensive follow-up with a variety of health care services. The health
care team decides that this is an appropriate client for case management based on which
of the following principles?
a)
The client has a limited support system.
b) The client is difficult to work with.
c) The health care team needs to work with this client.
d) The client requires a variety of services and resources.
Ans: d

425. A home health nurse has scheduled an interview for a new client. Before this visit,
however, the nurse receives a call from an established client, a prominent member of
the local community, who insists on seeing the nurse immediately, for a non-emergency
situation. The nurse explains that they will be able to make a visit, but it will be later in
the day. This nurse has addressed which concern of case management?
a) Beneficence
b) Non- maleficence
c) Equity
d) Autonomy
Ans: c

426. A community health nurse has been involved in the management of a client for
some time. The client was denied disability services after the nurse recommended this
option. The first action for the nurse, in this situation, is to:
a) Recommend that the client re-submit the application.
b) Make a formal request to the insurance company.
c) Call the physician.
d) Review all documentation and justification.
Ans: d

427. A community service, which includes food and clothing distribution to the needy or
homeless, was notified that funding would be dropped and the service will need to
become self-supporting. The decision was made at the city council level, without
strategies in place for future funding needs. This is an example of:
a) Breach of contract.
b) Wrongful denial of services.
c) Abandonment.
d) Negligence.
Ans: a

428. A community health nurse is assessing a client in the need for case management.
Which of the following would be an example of a personal indicator in this client's
background?
a) Borderline personality disorder
b) Single parent
c) History of anorexia
d) Recent drug overdose

Ans: a
429. A home health case manager is delegating tasks to even out the workload among
employees. Of the following, which task would be the most appropriate to delegate to an
unlicensed personnel?
a) Visiting a client who has been having problems with physical and cognitive decline.
b) Helping a new client with medication administration.
c) Selecting appropriate assistive devices for a client with sensory decline.
d) A home visit to a hospice client who needs assistance with bathing.
Ans:d
430. A community health nurse is evaluating a health program and its effects on the
population. The best indicator that this program acted as an empowerment to the
community is that now the community:
a) Provides disadvantaged groups within the population a voice in decision making.
b) Is successful in changing the social and political environment to improve equity and
quality of life.
c) Promotes group action.
d) Is rejuvenated in its physical and business structures.
Ans: b

431. A community health nurse holds an educational session for a group of clerks at a
local furniture store that carries a variety of car seats, strollers, and baby furniture. The
nurse's goal is for these employees to better inform parents and the public about child
safety. This group of clerks will now function in the capacity of:
a) Role-related helpers.
b) Volunteers.
c) Natural helpers
d) Community health workers
Ans: a

432. A community health nurse is making an assessment of a particular community.


Noting that there are two hospitals within the community, one public and one "not for
profit," would be examples of which level of assessment?
a) Tertiary
b) Internal
c) Primary
d) Secondary
Ans: d
433. A school health nurse has been working to help develop an after-school program
for teen mothers. After identifying the need for such a program, the nurse then looks to
the individuals who would benefit, including but not limited to the teen mothers. This
nurse is focusing on which element of issue selection?
a) Constituency
b) Targets
c) Goals
d) Direction
Ans: a

434. A community health nurse has helped initiate a program that provides meals and
socialization for the senior population. The program now is run entirely by volunteer
members of the community. Which level of community participation does this
represent?
a) Citizen power
b) Token
c) Therapy
d) Partnership
Ans:a
435. A community health nurse is developing a mentoring program between senior
citizens and high school students. The nurse meets with the senior citizens to help
develop goals and purposes of the project. This is an example of which stage of group
development?
a) Operation
b) Orientation
c) Negotiation
d) Accommodation
Ans: b
436. A community health nurse has been working with a local community to build a
program to serve the homeless and help with job re-training. As part of this process, the
nurse has organized several information sessions to the community in order to help
raise awareness for the need as well as the necessary involvement of the community for
this type of service. Which role of community empowerment is this nurse working in?
a) Discussion
b) Drive
c) Discovery
d) Decision
Ans: c
437. A community health nurse has been helping to implement a new program with and
for the community. Of all the roles the community will assume in this project, which
will be most important to negotiate?
a) Leadership
b) Advocacy
c) Conflict resolution
d) Accommodation
Ans: a
438. The community health nurse is working with a family consisting of two parents and
no children living in the home. The nurse categorizes this family as a(n):
a) Nuclear.
b) Extended.
c) Nuclear dyad.
d) Conjugal.
Ans:c
439. A school health nurse is working with an after-school program that includes
students mostly from African-American backgrounds. Formulating an assessment from
national statistics, the nurse understands that a large percentage of these students are
being raised in a(n):
a) Nuclear family system.
b) Nuclear dyad family system.
c) Extended family system.
d) Single parent family system.

Ans:d
440. A community health nurse is making an assessment of a group of children in a
community day care setting, most of whom live in single-parent family settings. The
nurse makes a point to check with the children's teachers in regard to:
a) Parental involvement.
b) Specific health concerns.
c) Behavior problems.
d) Signs of sexual abuse.

Ans:c
441. A nurse is working with family groups and realizes there are a number of
remarried parents retaining joint child custody. The nurse documents these children
specifically as part of which family type?
a) Bi-nuclear
b) Nuclear dyad
c) Single parent
d) Stepfamily

Ans:a
442. A community health nurse is working with a family in which the parents lived
together for three years prior to marriage. The nurse realizes that this family is at
greater risk for:
a) Teen pregnancies.
b) Divorce.
c) Illness.
d) Living below poverty level
Ans:b

443. A home health nurse works specifically with elderly clients. When assessing family
types, the nurse understands that which of the following is true?
a) More women will have health problems.
b) Women will be healthier.
c) Few elderly women live alone.
d) Most of the elderly do not live alone.

Ans:a
444. A school nurse has been working with children of a blended family for several
months. At one meeting, the children raised the issue of how to make the transition
easier for themselves as well as their parents, and volunteered to implement several
ideas the nurse suggested. This is an example of:
a) Goal setting.
b) Input.
c) Throughput.
d) Output.
Ans: c

445. While completing a family assessment, the nurse is using information about stages
of the family, what is normal for this stage, and how this particular family has adapted
to the stage. This nurse is utilizing which theoretical model?
a) McGoldrick
b) Duvall
c) Erikson
d) Carter
Ans:b

446. A nurse educator requires the students to prepare a genogram of the clients they
cared for in home health. The nurse is utilizing which theoretical model?
a) Developmental
b) Structural-functional
c) Functional
d) Duvall's model
Ans:b

447. A community health nurse is assessing the physical and psychological health of
teens within a community. The best way to assess behavior problems and discipline
measures is for the nurse to:
a) Interview individual clients for this information.
b) Become culturally sensitive to these practices among the population of interest.
c) Interview local physicians about these practices.
d) Check with the school system for information about discipline and family practices
Ans: b

448. A community health nurse is working with a local hospital to provide services that
cover education and immunization for parents and preschool-age children. The nurse is
working at which level of prevention in this case?

a) Primary
b) Tertiary
c) Secondary
d) Health promotion level
Ans: a

449. A community health nurse is working with a local physician's group in providing
care for pregnant women and specifically addressing their nutritional needs. Most of
these women are from low-income situations and do not have health insurance. The best
way to ensure they have adequate nutrition during pregnancy is for the nurse to:

a) Refer them to the local WIC program.


b) Provide them with prenatal vitamins.
c) Refer them to the local food pantry.
d) Make sure they know which local shelters serve hot meals
Ans: a

450. A school nurse frequently has to address care at the primary, secondary, and
tertiary level. Which of the following is an example of secondary prevention care?

a) Monitoring ventilator settings for a student who is enrolled at a local grade school.
b) Providing hepatitis B immunizations to incoming kindergarten students.
c) Developing an education program for teen moms regarding available and appropriate
contraception methods.
d) Educating parents of preschool students about dental services available in the
community.
Ans: a

451. A school health nurse is working with parents of a child who has terminal cancer.
An example of care at the tertiary level of prevention is:

a) Connecting the parents with an online support group.


b) Teaching the parents about the side effects of the cancer drugs.
c) Educating the rest of the school staff about measure to decrease the risk for infection
in this child.
d) Monitoring the child's lab values to prevent infections.
Ans: a

452. The nurse is assessing a client and identifies in the chart that the client is
white/non-Hispanic. The nurse is addressing which aspect of the client?
a) Nationality
b) Ethnicity
c) Culture
d) Race
Ans: d

453. A student nurse took a job as a certified nurse assistant while still in school and
began to adopt practices of the facility, which did not always represent practice as
taught in school. This job, however, made the student feel more appreciated by the rest
of the staff. This student nurse is exhibiting which component of culture?
a) Cultural competence
b) Acculturation
c) Cultural sensitivity
d) Ethnicity
Ans:b

454. A community health nurse is making an assessment of a young, adult client from
India. In the nurses' understanding of genetic predisposition, particular attention will
be applied to which body system?
a) uild and body structure
b) Cardiac
c) Metabolic
d) Hematologic
Ans: b
455. A psychiatric nurse is working with a new admission, a client from another culture.
During the admission interview, this culturally competent nurse asks questions that go
beyond the bio-medical realm. This nurse is assessing for:
a) Culture-bound syndromes.
b) Disease causation.
c) Homeopathic practices.
d) Psychological problems
Ans:a

456. A community health nurse is conducting screenings on a group of refugees.


Although no physical problems were identified, the nurse decides a follow-up is
necessary regarding which component of this group's new surroundings?
a) Physiological variances
b) Psychological considerations
c) Environmental safety
d) Socio-economic challenges
Ans:: c

457. Many cultures implement religious practices and beliefs regarding physical health.
From the following, select statements that relate to positive healthful behaviors.
a) Disease is seen as the will of God.
b) Laying-on of hands.
c) Manipulation of impersonal powers.
d) Pilgrimage and charismatic healing.
Ans::b
458. It is determined that an interpreter is necessary to communicate information about
a client's illness. The best method for interpreting is the use of:
a) Telephone interpretation.
b) Family members.
c) Gestures and signs.
d) Bilingual staff.
Ans:d

459. A community health nurse is working with a group of residents in an older part of
the city. Because many of them live in older buildings, which of the following should the
nurse include in the checklist for a safe environment?
a) CO2 levels
b) Fertilizer run-off
c) Pesticide levels in the water
d) Lead levels in the water
Ans: d

460. A client visits the community health office reporting frequent GI distress and states
that even though she has seen numerous physicians, no one has been able to identify a
cause of the problem. With an understanding of environmental health, the nurse
questions the client regarding:
a) Quality of air in their environment.
b) The level of noise in the workplace.
c) Exposure to pesticides.
d) Exposure to radiation.
Ans:b

461. A community health nurse is working with city planners to implement a variety of
environmental health activities at the primary level. Which of the following would be
appropriate?
a) Working with city council members to set up a program for radon screening
b) Screening for high levels of lead
c) Making referrals to physicians for chemical exposure testing
d) Assisting in the development of a walking trail
Ans::d

462. Working in the secondary level of prevention for the betterment of environmental
health, the nurse would:
a) Set up classes to teach community members how to conduct wellness exercise
sessions.
b) Provide assistance to apartment dwellers to remove lead-based paint from their walls.
c) Help organize a city-wide campaign for sidewalk improvement.
d) Lobby at the state level for increased funding for wellness programs

Ans:: b
463. A nurse is working with a family to help it adjust to a child's developmental delays
following exposure to lead. This would be an example of which level of prevention?
a) Tertiary
b) Secondary
c) Health promotion
d) Primary
Ans::a

464. A community health nurse has developed an educational program with a local pest
exterminator company. One of the objectives is to help consumers identify ways to keep
their homes pest-free. This is an example of
a) Secondary.
b) Primary prevention.
c) Tertiary.
d) Health promotion
Ans:: b

465. A community health nurse is seeing the child of a family who recently moved into a
new house. The child has been experiencing problems with rash and respiratory
symptoms. The nurse should assess which aspect of the environment?
a) Animals or pets
b) Plants and foliage
c) Climate or terrain
d) Structural environment
Ans:b

466. Following a winter of heavy snow, a public health nurse is concerned with the
safety of a community's water supply after unseasonably warm weather. The nurse
monitors the community's health for several months. Rationale for this concern is based
on the knowledge that:
a) Urban sewage contaminates drinking water.
b) Insect vectors breed in water.
c) Water can be contaminated with hepatitis A virus.
d) Groundwater contamination occurs within two months.
Ans: d

467. A public health nurse is making an assessment of the overall health of a


community. After looking at factors in the community that affect health, the next step
would be to:
a) Plan interventions.
b) Set goals for the health of the community.
c) Identify the population age groups.
d) Evaluate the interventions.

Ans:: c
468. A nurse educator is explaining the details of health care programs and relates that
they often can be a benefit for the state because they:
a) Are considered investments.
b) Offer increased productivity.
c) Allow states to control their own spending.
d) Generate more revenue than they use to.

Ans: d
469. A nurse educator teaches students that the reason health care costs have risen so
severely in the United States is because of the actual cost to health care providers for
malpractice insurance and expensive technology. The nurse is using which theory of
health care spending?
a) Per-capita spending
b) Demand-pull
c) Cost-push
d) Uncompensated care

Ans: c
470. A school nurse works with several children who have problems controlling their
asthma. The nurse realizes these children are raised in poverty and have limited access
to health care, which usually is provided by emergency room physicians. This is an
example of:
a) Poverty.
b) Negligence
c) Health disparity
d) Lack of insurance

Ans: c
471. A nurse is working with a client who has no prescription coverage, but requires
several prescribed medications to control a cardiovascular condition. The nurse
understands that this client will need to utilize which approach to financing health care
services?
a) Government
b) Self-funded
c) Personal payment
d) Commercial

Ans: c
472. A client visits the community health office and asks for assistance in finding
appropriate and affordable health care. The client is a pregnant, single mother. The
nurse understands that this client would fall into which category for health care?
a) Dual eligibility
b) Low income
c) Medically needy
d) Categorically needy

Ans: d
473. A parent, who secured a new job, visits the community health office concerned
about losing Medicaid benefits, and is particularly worried about her two children, who
have asthma. The parent has been out of work for some time and worked hard to find
employment. What is the best response by the nurse?
a) "Why don't you wait until another job comes along that has better benefits?"
b) "Let me tell you about the Children's Health Insurance Program."
c) "Have you asked your employer about insurance benefits?"
d) "Maybe you should re-think taking this low-paying job."

Ans: b
474. A client visits a local clinic and besides the initial assessment, blood is drawn for a
complete blood count as well as a chest x-ray. The bill arrives and they are billed for the
initial visit, a radiologic exam, and a laboratory test. This is an example of:
a) Fee-for-service reimbursement.
b) Prospective reimbursement.
c) Per diem payment.
d) Discounted fee-for-service mechanisms.

Ans: a

475. Community health nurses can be actively involved in making positive changes and
trends in the financial approach to health care services. An appropriate activity in this
regard would include:
a) Assisting practicing nurses with finding affordable malpractice insurance.
b) Promoting higher salaries for registered nurses in acute care facilities.
c) Assisting employers with ways to control their insurance premiums.
d) Research work in the area of health care delivery.
Ans: d
476. A home health nurse is working with a client who has several open wounds. Soiled
bandages from dressing changes, and equipment used for this client should be:
a) Handled as regular garbage, but marked as "Hazardous Waste."
b) Handled in the same manner as if the client were in a clinical setting.
c) Disposed of in the same manner as the client's garbage.
d) Brought to the local landfill so as not to mix with the client's regular waste.
Ans: b
477. A home health nurse is evaluating the effectiveness of the visits to a particular
client. Which of the following reflects evaluation of a primary preventive measure?
a) The client's weight after nutrition therapy was instituted.
b) The client's mobility and range of motion measurements after physical therapy
sessions.
c) The client's blood pressure measurements after initiating treatment for hypertension.=
d) The number of times the client attended health education classes.
Ans:d

478. A home health nurse is making a visit to a client whose dwelling does not have
running water. The best way for the nurse to practice standard precautions in this case
would be to:
a) Make a referral to social services, as this client should not be living in a home without
running water.
b) Only perform non-invasive procedures or assessments.
c) Have the client meet the nurse at the closest health agency.
d) Use waterless hand cleaner before, during, and after the visit.

Ans:d
479. A home health nurse visits a client who was referred following hospitalization for
severe malnutrition related to gastroesophageal reflux disease (GERD) and a large
hiatal hernia. Of the following, which statement reflects evaluation of a secondary
prevention measure? The client
a) Gained two pounds in the past month.
b) Relate the importance of taking a daily vitamin.
c) Will relate the importance of maintaining diet high in nutrient quality.
d) Can state the side effects of her medications.

Ans:: a
480. A home health nurse is making an initial visit to a new client. The nurse had
developed a preliminary assessment prior to the visit to the client, who was referred
after hospitalization for a total hip replacement. The client lives alone. Of the following,
which would be the next step in the home visit? The nurse will:
a) Monitor the client's progress in mobility.
b) Evaluate the home for any mobility concerns (i.e. stairs, carpets, uneven sidewalk,
etc.).
c) Refer the client to physical therapy.
d) Validate the information collected prior to the visit.
Ans:d

481. A home health nurse is visiting an established client who was referred following
hospitalization for a myocardial infarction. The client lives with a spouse and the nurse
is assessing the client's knowledge regarding resuming normal activities. The nurse has
been trying unsuccessfully to gain information about the client's resumption of sexual
activity, but the client consistently changes the subject to questions about medications.
This is an example of a(an):
a) Nurse-oriented distraction.
b) Behavioral distraction.
c) Physical distraction.
d) Environmental distraction
Ans:: b

482. A home health nurse is completing an initial visit to a client. Of the following,
which would be the most important to document?
a) The client's physical assessment.
b) The objectives developed by the nurse prior to the visit.
c) Discharge summary.
d) Referrals made to outside agencies.

Ans:a
484. A client has been receiving home health care for approximately four weeks. The
physician has referred the client to a pulmonary rehab program because of
complications of chronic obstructive pulmonary disease (COPD). The client is an 87-
year-old widow with various other chronic conditions and tells the nurse that she won't
be going to rehab because it seems more bother than what it will be worth. The nurse
should:
a) Make sure the client's family will persuade her to consider rehab.
b) Convince the client that rehab would be in her best interest.
c) Tell the client that her physician has determined that this is the best course of action.
d) Let the physician know the client won't be going to rehab.

Ans:d
485. A hospice client has decided to remain in his own home until death. The client
requires 24/7 nursing care and during the first overnight care by hospice nursing staff,
they realize that the client's home is infested with cockroaches. The hospice nurse
should:
a) Make an immediate referral for social services and move the client to a long-term care
facility.
b) Refuse to visit the home until the situation is taken care of.
c) Tell the client's family that the client does not qualify for hospice services because of
the condition in the home.
d) Help the family make arrangements for pest control measures immediately.
Ans: d
486. A nurse is evaluating the clients of a nursing home. In addition to assessing their
physical needs, the nurse understands that many of these clients are most likely in
various stages of life review. The nurse is utilizing which theory?
a) Biological
b) Psychological
c) Sociological
d) Developmental
Ans: d
487. A community health nurse volunteers at a local senior citizen center several times a
week. As the participants come in for weekly social activities, the nurse understands
that according to statistics:
a) Less than half the women will report some type of functional disability.
b) There will be no difference in mobility between the men and women.
c) Men will be more apt to use assistive devices.
d) Women will be less mobile than men.
Ans: d
488. A client with advanced arthritis is being seen in the outpatient clinic. The nurse
takes a thorough medication history and notes that the client does not seem to be in
pain, merely by observation (i.e. no grimacing or guarding). The nurse can correctly
make which of the following conclusions?
a) The physician will be able to appropriately treat the client's discomfort.
b) The client is not in pain since there are no outward signs suggestive of discomfort.
c) The client's medications are appropriately dosed.
d) The client's pain may or may not be well-controlled.
Ans: d

489. A client has been diagnosed with early stage Alzheimer's disease. As an
appropriate intervention, the home health nurse recommends:
a) Finding a suitable long-term care facility for the client.
b) Feeding the client to assure adequate nutrition.
c) Keeping the client in a regular routine.
d) Arranging for the client to have new activities and not become bored.

Ans: c
490. A community health nurse is working with a family whose elderly parent lives in
the home with the married child and two dependent children. As caregivers to the
parent, the clients voice concerns over the recent and sudden behavior changes they've
noticed (i.e. forgetfulness, confusion, and weakness). The nurse, using knowledge of the
elderly population, should respond:
a) Does Alzheimer's disease run in your family?"
b) "What kind of medications is your parent taking?"
c) "Do you pay attention to how much liquid your parent takes in 24 hours?"
d) "Confusion is a normal sign of aging."

Ans: : c
491. A group of clients at a senior citizen center are engaged in an exercise class three
times per week. As time goes on, one of the clients states to the nurse that they're
thinking of quitting since they really haven't seen any weight-loss benefit from the
classes. The best response by the nurse is:
a) Exercise will help prevent falls."
b) "Why would you want to miss seeing your friends?"
c) "Maybe we should look at your diet and nutrition."
d) "What kinds of foods do you eat after exercise class?"
Ans: a

492. A home health nurse is visiting an elderly client after a long rehabilitation course
following hospitalization for congestive heart failure. The client states that even though
he feels well enough,

a) Have you talked to your wife about this?"


b) "Decreasing sexual drive is just part of getting older."
c) "Let's have a look at all your medications."
d) "Well, after being in the hospital, you probably shouldn't be thinking about having
sex."
Ans: c
493. A community health nurse is working with elderly clients who requested help with
being able to make more educated decisions regarding their health care. One way the
nurse can assist with this is to:
a) Make sure the clients are aware of physicians who have a bad reputation with dealing
with the elderly.
b) Provide opportunities for the clients to rehearse and role play before seeing their
primary care physician.
c) Bring an adult child along so all information is heard accurately.
d) Remind the clients to tell their primary care providers to speak loudly and clearly.
Ans: b
494. A community health nurse is implementing an education series focusing on
nutrition and exercise for elderly clients. Of the following, which would be most
appropriate for the nurse to implement? The sessions should be:
a) All day every Saturday.
b) Daily, right after lunch, until late afternoon.
c) Every morning, for approximately 15 minutes.
d) Every other evening.
Ans: c

495. A client with gestational diabetes asks the nurse if she will have diabetes after the
baby is born. The correct response by the nurse is
a) "You have a greater risk to develop type 2 diabetes."
b) "Don't worry yourself about that, you need to concentrate on having a healthy baby."
c) "Yes, diabetes is a chronic disease of which there is no cure."
d) "Once you have the baby, your diabetic symptoms will disappear."
Ans: a

496. A community health nurse is working in a poverty area where most of the women
do not have health insurance. Understanding that Medicaid coverage provides for this
in all 50 states, the nurse stresses the importance of which activity?
a) Cervical and breast cancer screenings
b) Cardiac and hypertension screenings.
c) Yearly Pap smears.
d) Yearly annual physicals.
Ans:a

497. A nurse is working with a lesbian client. Which statement by the client would
indicate a need for further teaching? "Because I am in a homosexual relationship, I
a) "... will need to be screened for cervical cancer."
b) "... should notify my partner(s) if I test positive for any sexually transmitted disease."
c) "... will get a mammogram every two years."
d) "... do not need to be screened for STDs."
Ans:d

498. As a community health nurse working with lesbian clients, it is important for the
nurse to understand that these clients:
a) Will always be at risk for depression.
b) Are at no greater risk for psychiatric disorders.
c) Have no problems with psychological problems.
d) Are at greater risk for psychiatric disorders.
Ans: d

499. The community health nurse staffs a clinic, which serves a large number of
homosexual clients. When assessing young lesbian clients, the nurse would want to pay
keen attention to physiological changes of which system?
a) Integumentary
b) Genito-urinary
c) Respiratory
d) Musculoskeletal
Ans: c
500. A community health nurse is presenting a seminar during a "women's health
weekend" at a local clinic. The topic is "Mom's Juggling Act" and an objective is to
detect stress buildup and burnout. This is an activity aimed at which level of
prevention?
a) Primary
b) Health screening
c) Tertiary
d) Secondary
Ans:a
501. A nurse is working with an abused client. Which of the following would be an
example of secondary prevention for this client?
a) Assisting the client with making a list of "safe" places she can call in emergencies.
b) Making sure that the client feels worthy of respect
c) Assisting the client in finding a suitable counselor.
d) Assisting the client with job re-training.
Ans:b
502. A school nurse is planning an education session for an after-school program and
wants to incorporate learning that is demonstrated in the affective domain. The nurse
should have the students:
a) Take a test over the material at the end of the session.
b) Prepare and present a speech about the objectives of the session.
c) List and explain the important aspects that are outlined in the objective section of the
program.
d) Identify and role play positive communication skills with each other.
Ans: d
503. A nurse educator is explaining the concept of mainstreaming students within a
school system and asks the nursing students to provide an example of how this makes a
school nurse's job difficult at times. The best example is:
a) Making sure the teachers within the school system understand the role of the school
nurse.
b) The nurse caring for a child on a ventilator in the school system would have very little
time devoted to screen the rest of the students for vision problems.
c) Having time to get to know the students, their particular situations, and understanding
a little bit about their individual backgrounds.
d) Addressing the screening needs of several hundred students in three different
buildings with a limited amount of time.
Ans:b
504. A school system is interviewing candidates for the position of school nurse. Of the
following, which candidate would be most likely selection for the position? The
candidate who is:
a) An LPN.
b) Any RN
c) A pediatric nurse practitioner.
d) An RN with a baccalaureate degree
Ans:d
505. A school nurse is presenting an orientation for unlicensed assistive personnel
(UAP) that will be working with juvenile diabetics within the school system. Among the
many things that need to be addressed, the most important concept for each UAP to
grasp is:
a) Understanding the effects of long-term hypoglycemia.
b) Identifying changes in behavior and affect.
c) Learning how to give insulin.
d) Recommending that the child have yearly eye exams
Ans:b

506. The school nurse's role can be quite diverse in the number of other employees they
will eventually work with. An important in-service the nurse might provide to the
janitorial and custodial staff would include:
a) Classes on weight reduction and/or smoking cessation.
b) Information on disease transmission.
c) Stress reduction and coping skills for a healthier lifestyle.
d) The benefits of receiving a flu vaccine.
Ans:b
507. An alumnus of a particular school system has returned as the school nurse.
Assessing the community at large allows this nurse to identify major changes in not only
the student population but the outlying geographic area as well. This is an example of
which level of the school's ecological system as it relates to this particular nurse?
a) Mesosystem
b) Chronosystem
c) Exosystem
d) Macrosystem
Ans:b
508. A school nurse is working with a student who, up until this time, had been well-
behaved and exhibited no behavioural problems or concerns to the parents. Now, the
parents relate that the child is sleeping poorly, has lost weight, and seems to be sick
from school more often than what was normal in the past. After determining that there
is a good parental and peer relationships present for this child, the nurse should:
a) Spend time with the student and their friends.
b) Refer the student to a psychiatrist.
c) Have the student come in for a thorough psychiatric evaluation.
d) Ask the child's teacher about any noticed or identified problems
Ans:d
509. A school nurse has just begun working for a school system with a reputation for
high numbers of expulsions. The first thing the nurse should do is:
a) Notify the police.
b) Call social services to report the school's record.
c) Set up a meeting with the school administration and parents.
d) Review school policy
Ans:d
510. Infectivity of chickenpox last for:
a) 6 days after appearance of rash
b) 3 days after appearance of rash
c) Till the last scab fall offs
d) As long as fever last

Ans:a

512. Which of the following diseases gives life-long immunity after an attack?
a) Typhoid
b) Tetanus
c) Mumps
d) Diphtheria

Ans:a
513. Smallpox was eradicated because of the following factors except:
a) Potent vaccine
b) Easy to recognize
c) Extra human reservoir control
d) Long incubation period

Ans:c
514. Koplik's spots are seen in:
a) Chickenpox
b) Typhoid
c) Rubeola
d) Rubella

Ans:c
515. All stages of rash are seen in:
a) Smallpox
b) Typhoid
c) Measles
d) Chickenpox

Ans:d
516. Carriers are associated with transmission of disease in all except:
a) Typhoid
b) Measles
c) Cholera
d) Diphtheria
e) Ans:b
517. Sub-acute sclerosing pan encephalitis (SSPE) can occur in infection with:
a) Rubella
b) Mumps
c) Measles
d) Chickenpox
Ans:c
518. Which is not true about measles:
a) Secondary attack rate of 30%
b) Sub clinical infection
c) More severe in malnourished
d) Incubation period of 10 to 14 days

Ans:a
519. Which of the following is true of chickenpox?
a) Virus is not found in scab
b) Virus can be grown on the chick embryo
c) Caused by RNA virus
d) Does not cross the placental barrier

Ans:a
520. Recommended site of administration of DPT in infants is:
a) Lateral aspect of thigh
b) Deltoid
c) Forearm
d) Gluteal

ans:d
521. which is not Modifiable risk factors in case of hypertension?
a) Salt intake
b) Alcohol intake
c) Age
d) Obesity

Ans:c
522. Major epidemics of influenza A occur at intervals of:
a) 2-3 years
b) 1 year
c) 10-15 years
d) 20-25 years

Ans:d
523. The diseases which is not notifiable to the WHO under the International
Regulation Act are:
a) Cholera
b) Diphtheria
c) Yellow Fever
d) Plague

Ans:b
524. The vaccine usually given to pilgrims to the Middle East country is:
a) Yellow fever
b) meningitis
c) DPT
d) Influenza
Ans:a
525. Schick test is done in:
a) poliomyelitis
b) meningitis
c) pertussis
d) diphtheria

Ans:d
526. Infections primarily of animals that cannot be transmitted through milk is:
a) Q fever
b) Malaria
c) Tuberculosis
d) Brucellosis

Ans:b
527. Quickening means:
a) Nausea and vomiting in the morning
b) Hearing of the foetal heart sound
c) Sudden cessation of Menstruation
d) The movement of the foetus in the uterus felt by the mother

Ans:a
528. Period of maximum infectivity in mumps is during :
a) after swelling appears
b) prodromal period
c) onset of swelling
d) incubation period

Ans:c
529. A retrospective attempt to determine the cause of an event is:
a) Root cause analysis.
b) External benchmarking.
c) Internal trending.
d) Tracer methodology.

Ans:a

530. Which of the following is not considered as Metaparadigm of Nursing?


a) Nursing
b) Environment
c) Person
d) Diagnosis
e) Health
Ans:d

Das könnte Ihnen auch gefallen